You are on page 1of 152

ADRIAN STAN

Dreptul de copyright: Cartea downloadat de pe site-ul www.mateinfo.ro nu poate fi publicat pe un alt site i nu poate fi folosit n scopuri comerciale fr specificarea sursei i acordul autorului

Refereni tiinifici:

Prof. gr. I Mnzal Iorgu - inspector de matematic Inspectoratul colar Judeean Buzu Prof. gr. I Stanciu Neculai director Grupul colar Tehnic Sf. Mc. SAVA, Berca

BREVIAR TEORETIC

DIVIZIBILITATE
Relaia de divizibilitate: a b (sau ba)

c N respectiv Z astfel nct a=bc.

(a se divide cu b) sau (a este divizibil cu b) sau ( b divide pe a) Proprieti: 1. aa , a Z * 2. 1a, a Z 3. ab, bc ac, a, b Z * 4. da,db da+b sau da-b 5. ab, ba a=b , a, b Z * 6. da dabc 7. ad, bd abd, dac a i b sunt prime ntre ele. Descompunerea n factori primi:

......... p k k N Numrul divizorilor lui n N este: N= (a 1 + 1 )(a 2 + 1 )........ (a k + 1 ) . Numrul divizorilor lui n Z este: N=2N
a a2 a3 a

n = p1 1 p 2

p3

Numere prime Numim numr prim orice numr natural mai mare dect 1, care are numai divizori improprii-adic pe 1 i pe el nsui.

Criterii de divizibilitate:

Criteriul de divizibilitate cu 2 Un numr este divizibil cu 2 dac ultima sa cifr este par. Numerele care sunt divizibile cu 2 se numesc numere pare. Criteriul de divizibilitate cu 5 Un numr este divizibil cu 5 dac ultima sa cifr este 0 sau 5. Criteriul de divizibilitate cu 4 Un numr este divizibil cu 4, dac numrul format de ultimele sale 2 cifre este divizibil cu 4. Criteriul de divizibilitate cu 8 Un numr este divizibil cu 8, atunci cnd n umrul format de ultimele sale 3 cifre este divizibil cu 8. Criteriul de divizibilitate cu 25 Un numr este divizibil cu 25, dac numrul format de ultimele sale 2 cifre este divizibil cu 25, adic dac ultimele sale 2 cifre sunt:00;25;50; 75. Criteriul de divizibilitate cu 125 Un numr este divizibil cu 125, dac numrul format de ultimele sale 3 cifre este divizibil cu 125. Criteriul de divizibilitate cu o putere a lui 10 Un numr este divizibil cu o putere a lui 10, dac ultimele sale n cifre sunt zerouri. Criteriul de divizibilitate cu 3 Un nr.este divizibil cu 3, dac suma cifrelor sale este un numr divizibil cu 3. Criteriul de divizibilitate cu 9 Un numr este divizibil cu 9, daca suma cifrelor sale este divizibil cu 9. Criteriul de divizibilitate cu 6 Un numr este divizibil cu 6, dac este divizibil cu 2 i cu 3. Criteriul de divizibilitate cu 15 Un numr este divizibil cu 15, dac este divizibil cu 5 si cu 3. Criteriul de divizibilitate cu 11 Un numr este divizibil cu 11, dac diferena dintre suma cifrelor situate pe locurile impare si suma cifrelor situate pe locurile pare este un numr divizibil cu 11. Teorema mpririi cu rest n N.

Fie a, b N q, r N ,0 r b a. a = b q + r , b 0 Cel mai mare divizor comun al numerelor a i b (c.m.m.d.c) sau (a,b) este cel mai mare numr la care se mpart exact si a si b i este dat de produsul factorilor comuni, luai la puterea cea mai mic. 1) (a;b)=d <=> a=dxa', b=dxb', (a';b')=1 2) (a;b)=d <=> d/a si d/b, oricare ar fi d' a.. d'/a si d'/b=> d'/d Cel mai mic multiplu comun al numerelor a si b (c.m.m.m.c.) sau [a,b] este cel mai mic numr care se mparte exact i la a i la b i este dat de produsul factorilor comuni i necomuni luai la puterea cea mai mare. 1)[a;b]=m <=> m=axm' , m=bxm' 2)[a;b]=m <=> a/m si b/m, oricare ar fi m', a.i. a/m' si b/m'=>m'/m

Relaia dintre c.m.m.m.c i c.m.m.d.c


[a,b](a,b)=ab Dac p i q sunt prime atunci p n si q m sunt prime.

MULIMI. OPERAII CU MULIMI

N * = N \ {0} Z = {...... n,.... 3,2,1,0,1,2,3,........., n,.......} Z * = Z \ {0} a Q = a, b Z , b 0 Q* = Q \ {0} b R \ Q = n n 0 nu este patrat perfect R = Q ( R \ Q) = (,+) .
N = {0 ,1, 2 ,3,......... .. n ,...... }

Fie A i B dou mulimi. Atunci:

A B = x x A sau x B . A B = x x A si x B . A \ B = x x A si x B . AxB = ( x, y ) x A si y B .

Reuniunea mulimilor. Intersecia mulimilor. Diferena mulimilor. Produsul cartezian.

Definiie: Se numete cardinal al unei mulimi finite , numrul de elemente pe care-l are aceasta. Principiul includerii i excluderii: card(A B) =card (A) +card(B) card (AB). Definiie: Se numete submulime al unei mulimi A, orice mulime format cu elementele lui A. Numrul tuturor submulimilor unei mulimi cu n elemente este 2n. FRACII Fracie: Se numete fracie, o expresie de forma numete numrtor iar b se numete numitor. Fracie subunitar: Fracia dac a<b sau

a , b 0 unde a se b

a , b 0 se numete fracie subunitar b

a 1 . b a , b 0 se numete fracie echiunitar b

Fracie echiunitar: Fracia dac a=b sau

a =1. b

Fracie supraunitar: Fracia supraunitar dac a>b sau

a , b 0 se numete fracie b

a 1 . b a c si se numesc echivalente i b d

Fracii echivalente: Dou fracii scriem

a c = dac a d = b c . b d

A amplifica o fracie cu un numr natural, diferit de 0, nseamn a nmuli att numrtorul ct i numitorul, cu acel numr.

a am = , b 0, m 0. b bm

A simplifica o fracie cu un numr natural, diferit de 0, nseamn a mpri att numrtorul, ct i numitorul la acel numr.

a a:m = , b 0, m 0. b b:m
Fracie ireductibil: Fracia

a , b 0 se numete b

ireductibil,

dac nu se mai poate simplifica adic c.m.m.d.c(a,b)=1. Compararea fraciilor:

a c a c b b
Numr raional :

a a b c b c

a c a d b c b d a i b

Se numete numr raional , numrul reprezentat prin fracia toate fraciile echivalente cu aceasta.;

Operaii cu numere fracionare: Pentru a aduna sau scdea numere fracionare reprezentate prin numitori diferii se aduc fraciile la

acelai numitor prin amplificarea fiecrei fracii cu ctul dintre c.m.m.m.c al numitorilor i numitorul fraciei respective.

a c a + c a c ac a c a d ; ; + = : = ; = b b b b d bd b d b c 0 m n m+ n a a a a = 1; = ; b b b b
m n mn
n

mn 1 a m b a a ; = ; = a b b b b ac + b ; Introducerea unui ntreg n fracie: a = c c

a a a : = b b b

Fracii zecimale: - Fracii zecimale: finite: {0,5;1,45;2,5;0,12345;......} Transformarea fraciilor zecimale n fracii ordinare:

a, b =

1 - fracii zecimale -periodice: { , (3);21,3(5);2,3( 4)......} Transformarea fraciilor zecimale periodice n fracii ordinare:

ab ; 10

a, bc =

abc ; 100

ab a abc a ; a, (bc) = ; 9 99 abc ab abcd ab a, b(c ) = ; a, b(cd ) = ; 90 990 a, (b) =


Scrierea n baza zece:

abcd = a 103 + b 102 + c 10 + d


sutelor; c-cifra zecilor; d-cifra unitilor;

a-cifra miilor;

b-cifra

a, efg = a 10 + e 101 + f 102 + g 103 = = a 10 + e 0.1 + f 0.01+ g 0.001


a-cifra unitilor, e-cifra zecimilor; miimilor. f-cifra sutimilor; g-cifra

Aflarea unei fracii dintr-un numr :

a a din x = x ; b b

a c a c ac din ; = = b d b d bd

Procente: Un raport n care numitorul este 100, se numete raport procentual si se noteaz de forma

p 1 = p 0 0 ; (sau p ) 100 100 a p a 1 a sau p = b 100 b 100 b

p 0 0 din x =

p x; 100

p 0 0 din

Operaii cu fracii zecimale: La adunarea sau scderea fraciilor zecimale finite, numerele trebuie aezate astfel nct virgula s fie sub virgul. La nmulirea cu 10, 100, 1000 a unei fracii zecimale finite se deplaseaz virgula spre dreapta cu 1,2,3 cifre. La mprirea cu 10,100,1000, a unei fracii zecimale finite se deplaseaz virgula spre stnga cu 1,2,3 cifre. La nmulirea fraciilor zecimale finite, efectum nmulirea obinuit dup care punem virgula de la dreapta spre stnga dup un numr de zecimale egal cu numrul de cifre zecimale ale celor dou numere; La mprirea fraciilor zecimale finite se vor nmulii ambele numere cu puteri ale lui 10 astfel nct s mprim numere fr virgul.

Ultima cifr a unui numr


U(cn) 1n 2n 3n 4n 5n 6n 7n 8n 9n

U (abc ) = U (c n )
n=4k+2 1 4 9 6 5 6 9 4 1 n=4k+3 1 8 7 4 5 6 3 2 9 n=4k 1 6 1 6 5 6 1 6 1

n=4k+1 1 2 3 4 5 6 7 8 9

Proporii: Egalitatea a dou rapoarte se numete proporie: Proprietatea fundamental a proporiilor: Proporii derivate:

a c = a d = bc b d

a c a b ab cd a b = , = = = b d ba d b b d c d 2 2 a c a c a a+c = = 2 = 2 b:m d :m b b+c b d am cm a c a a+c = = = b d b:m d :m b b+c am cm = b d


a ca = b d b

a c = b:m d :m

10

Sir de rapoarte egale: Mrimile (a1,a2,a3,......, an ) i ( b1,b2,....,bn ) sunt direct proporionale

a a1 a 2 = = ....... n b1 b2 bn

(b1,b2,....,bn ) sunt invers Mrimile (a1,a2,a3,......, an ) i proporionale a1 b1 = a 2 b2 = .....a n bn

Probabiliti
Probabilitatea realizrii unui eveniment este dat de raportul dintre numrul cazurilor favorabile realizrii evenimentului i numrul cazurilor egal posibile. Modulul numerelor reale Proprieti:
a, 0, a, a0 a = 0 a 0

def

1. a 0, 3. a = a ,

a R ;

2. a = 0, 4. a = b , 6.

a = 0;

a R ;

a = b ;

5. a b = a b ; 7.

a a = ; b0 b b

, x R , n N

8.

a b a + b , a, b R

9. a b a b a + b ; a, b R

11

10. x = a, 11. x a, 12.

x = a,

a 0 ; a 0 ;
, a, b R ;

x [ a, a ],

max (a, b) = min (a, b) =

a+b+ ab
2 a+b ab
2

13. x a,

x [, a ] [a,+],

a 0 ;

14. a1 a 2 ... a n a1 + ... + a n , in R .


Fie a R, n Z

Puteri cu exponent ntreg

a n def

a a a ...... a
n factori

Fie a, b R, a, b 0, m, n Z *

1. a o = 1; a 1 = a;0 n = 0; 2. a m + n = a m a n 3. ( a b ) = a b
n n n

5 . ( a m ) n = a m n 6. a n =
n

1 an

an a 7. = n b b 8. a m = a n m = n.
n

am 4. n = a m n ; a

1, n par 9. (-1) = 1, n impar


n

a 10. b

b = a

12

Proprietile radicalilor

Fie a , b R
1. 2. 3. 4. 5.

a 2 = a 0,

a b = a b a b
n

a , b0 b
n n 2

a = ( a) = a a b =

a + a2 b a a2 b 2 2
(formula radicalilor dubli)

unde a-b=k . 6. Dac 7. a b = 8.

a N a = k2
a 2b

a 2b = a b

9. x a y b = xy ab 10. x a y a = ( x y ) a 11.

x a y b

=
n

12. x a 13. Fie

( )

x y

a , y, b 0 b

= xn an a= m Q m = n = 0 n

a R \ Q . Atunci

14. Dac m, n Q i a nu e ptrat perfect i

m+n a =0m= n =0 15. Dac a, b N i a + b Q a N , b N


16. Dac a i b nu sunt ptrate perfecte

a + b Q

13

Q+ i , Q a.. a + b Q a Q , b Q 18. Dac a, b Q a.. b R \ Q +


17. Dac a, b
+

a b R \ Q i

a bR\Q
19. Dac a

i b R \ Q a + b R \ Q i

a bR \Q
Raionalizri

x a x

x a , a =

x a b

x b ab x = x( a + b ) a2 b

a+ b
Medii

x(a b ) , a2 b

a b

x+ y 2 Media geometric m g = x y , x, y R * +
Media aritmetic ma = Media ponderat m p =

px+q y ; p, q N * ponderile p+q 2 2 xy * Media armonic m h = , x, y R = 1 1 x+ y + x y

14

ECUAII

b a x + b = 0 a x = b x = , a 0. a 2 x = a x = a , a 0. ;
a x 2 + b x + c = 0 x1, 2 = b b 2 4ac 2a

a 0. b 2 4ac 0

[x] = a a x a + 1 x [a, a + 1)
16. PROCENTE

x = a, a 0 x = a.

p % din N =

Raportul

p N 100 p se numete raport procentual iar p se numete 100

procent. Aflarea unui numr cnd cunoatem p% din el.

p 100 din x =a x = a . p 100 Aflarea raportului procentual:


Ct la sut reprezint numrul a din N ? p % din N =a p =

a 100 . N

D=

S pn . Dobnda obinut prin depunerea la banc a unei 100 12

sume S de bani pe o perioad de n luni cu procentul p al dobndei anuale acordate de banc .

15

CALCUL ALGEBRIC Reguli de calcul n R 1. (a + b ) = a 2 + 2ab + b 2 ;


2

2. (a b ) = a 2 2ab + b 2 ;
2

4. (a + b + c ) = a 2 + b 2 + c 2 + 2ab + 2bc + 2ca


2

3. (a+b)(a-b= a 2 -b 2 ;
3

5*. (a + b ) = a 3 + 3a 2 b + 3ab 2 + b 3 ; 6*. (a b ) = a 3 3a 2 b + 3ab 2 b 3 ;


3

7*. a 3 b 3 = (a b)(a 2 + ab + b 2 ) ; 8*. a 3 + b 3 = (a + b)(a 2 ab + b 2 ) .

Descompuneri n factori
1. Metoda factorului comun

ab+ac-ad=a(b+c-d) ax+ay+by+by=a(x+y)+b(x+y)=(x+y)(a+b) a2+2ab+b2 =(a+b)2 a2-2ab+b2 =(a-b)2 a2+b2+c2+2ab+2bc+2ac=(a+b+c)2 a2-b2=(a-b)(a+b)


2. Utilizarea formulelor de calcul prescurtat

Rapoarte de numere reale reprezentate prin litere

Amplificarea

a ac = , b, c 0 b bc a a:c Simplificarea = , b, c 0 b b:c


16

Adunarea sau scderea nmulirea

a c ac = , b, d 0 b d bd a c a d mprirea : = b, c, d 0 b d b c a an Puterea cu exponent natural ( ) n = n , b 0, n N * b b Puterea cu exponent ntreg negativ a b


n

a c ac = , b0 b b b

b = , a, b 0, n N * . a

17

FUNCII Sistem de axe ortogonale

Definiie: Se numete funcie, un triplet de forma (A,B,f), unde A se numete domeniu de definiie (mulimea de unde funcia ia valori), B se numete codomeniu (mulimea valorilor funciei), iar f se numete lege de coresponden(face ca fiecrui element din A s-i corespund un unic element din B).

Notaie: f:AB

Imaginea funciei este mulimea Im f= y B y = f ( x), x A

18

Exemplu: f(x)=x2

Graficul unei funcii f:AB este mulimea G f = ( x, f ( x ) ) x A A B .

Condiia ca un punct M(a,b) s aparin graficului lui f. M ( a, b) G f f ( a ) = b .

Reprezentarea grafic: f: RR , f(x)=ax+b graficul este o dreapt;

f (x) not y

y=ax+b

OX : y = 0 x =

b b A( ,0) a a

OY : x = 0 y = b B(0, b)

19

f: IR, unde I este un interval; dac I este mrginit, atunci graficul este un segment iar dac I este nemrginit la un capt i mrginit la cellalt atunci graficul este o semidreapt.

f:AR, unde A este o mulime finit de puncte atunci graficul lui f


este tot o mulime finit de puncte. Condiia ca trei puncte A(x1,y1), B(x2,y2), C(x3,y3) s fie coliniare: - se determin funcia f(x)=ax+b, al crui grafic este determinat de dou puncte i se verific dac i cel de-al treilea punct aparine graficului lui f; - cu ajutorul lungimilor distanelor dintre puncte , se verific dac lungimea segmentului cel mai mare este egal cu suma lungimilor celorlalte dou segmente.

UNITI DE MSUR

Multiplii i submultiplii metrului- uniti de msur pentru lungime km hm dam


m

dm

cm

mm

Multiplii metrului

Submultiplii metrului

Multiplii i submultiplii m2- uniti de lungime pentru arie km2 hm2 dam2
m2

dm2

cm2

mm2

Multiplii metrului ptrat 1 ha =1 hm2=10000m2

Submultiplii metrului 2 1 ar = 100m2

20

Multiplii i submultiplii m3-uniti de msur pentru volum km3 hm3 dam3


m3

dm3

cm3

mm3

Multiplii metrului cub

Submultiplii metrului3

Multiplii i submultiplii litrului- uniti de msur pentru capacitate kl hl dal


l

dl

cl

ml

Multiplii litrului 1 dm3=1 l

Submultiplii litrului

Multiplii i submultiplii gramului- uniti de msur pentru mas kg hg dag


g

dg

cg

mg

Multiplii gramului 1 q=100kg Uniti de msur pentru timp

Submultiplii gramului 1 t = 1000kg

1 min=60 s 1 h =60 min=3600s 1 zi =24 h 1 an =365 zile sau 366 zile(an bisect) 1 deceniu = 10 ani; 1 secol =100 ani;

1 mileniu = 1000 ani.

21

UNGHIURI
Def 1. Dou unghiuri proprii se numesc opuse la vrf dac laturile lor formeaz dou perechi de semidrepte opuse. Def 2. Dou unghiuri se numesc complementare dac suma msurilor lor este de 90 i se numesc suplementare dac suma msurilor lor este de 180. Def 3. Dou drepte din acelai plan care nu au nici un punct comun se numesc drepte paralele. Postulatul lui Euclid: Printr-un punct exterior unei drepte trece o singur dreapt paralel la dreapta dat. Def 4. Dou drepte concurente sunt perpendiculare dac unul din unghiurile care se formeaz n jurul punctului lor comun este de 90. Def 5. Dou drepte paralele intersectate de o secant formeaz : - unghiuri alterne interne congruente: 4 cu 6, 3 cu 5; - unghiuri alterne externe congruente: 1 cu 7, 2 cu 8; - unghiuri corespondente congruente: 4 cu 8, 3 cu 7, 2 cu 6, 1 cu 5; - unghiuri interne sau externe de aceeai parte a secantei suplementare: 1 cu 4, 1 cu 8, 4 cu 5, 5 cu 8.

22

Def 6. Se numete unghi ascuit, unghiul a crui msur este mai mic de 90 . Def 7. Se numete unghi obtuz, unghiul a crui msur este mai mare de 90 . Def 8. Se numete unghi nul, unghiul a crui msur este de 0 . Def 9. Se numete unghi alungit , unghiul cu semidreptele n prelungire i cu msura de 180. Teorema 1. Suma msurilor unghiurilor n jurul unui punct din plan este de 360. Teorema 2. Suma msurilor unghiurilor n jurul unui punct pe o dreapt este de 180. Def 10. Dou unghiuri se numesc congruente dac au aceeai msur. Def 11. Dou unghiuri se numesc adiacente dac au vrful comun i o latur comun situat n interiorul unghiului format de celelalte dou laturi ale unghiurilor.

23

Triunghiul. Linii importante n triunghi. Aria triunghiului:

BC AD 2 AABC = p ( p a )( p b)( p c) AB + AC + BC (semiperimetru) unde p = 2 AB AC sin A AABC = 2 AABC =


nlimea este segmentul de dreapt ce unete vrful triunghiului cu piciorul perpendicularei dus din vrf pe latura opus; Intersecia nlimilor este un punct ce se numete ortocentru. Mediana este segmentul de dreapt ce unete vrful triunghiului cu mijlocul laturii opuse; intersecia medianelor este un punct ce se numete centru de greutate care se afl la o treime fa de baz i dou treimi fa de vrf.

24

Lungimea medianei n funcie de laturi:

AM 2 =

2( AB 2 + AC 2 ) BC 2 . 4

Mediana mparte un triunghi n dou triunghiuri echivalente, care au aceeai arie: AABM=AAMC Bisectoarea este segmentul de dreapt ce mparte unghiul triunghiului n dou unghiuri congruente. Intersecia nlimilor se noteaz cu I i se numete centrul cercului nscris n triunghi. Orice punct aflat pe bisectoarea unghiului se afl la egal distan de laturile unghiului. Mediatoarea laturii unui triunghi este dreapta perpendicular pe latura triunghiului exact prin mijlocul ei. Intersecia mediatoarelor se noteaz cu O i se numete centul cercului circumscris triunghiului. Orice punct aflat pe mediatoarea laturii unui triunghi se afl la egal distan de capetele segmentului. Linia mijlocie n triunghi este segmentul de dreapt ce unete mijloacele a dou laturi ale triunghiului; Linia mijlocie este paralel cu baza i jumtate din ea.

Teorema liniei mijlocii: MN-linie mijlocie MN BC , MN =

1 BC 2

25

Teorema reciproc asupra liniei mijlocii: Dac AM MB i

MN BC AN NC , MN =

1 BC 2

Dac MN este linie mijlocie n triunghi atunci mijloacele nlimii, bisectoarei i medianei aparin liniei mijlocii; Metoda triunghiurilor congruente: Pentru a arta c dou segmente sau dou unghiuri sunt congruente trebuie artat c triunghiurile din care fac parte sunt congruente. Cazurile de congruen ale triunghiurilor oarecare: I. L.U.L. II. U.L.U III. L.L.L. IV. L.U.U Cazurile de congruen ale triunghiurilor dreptunghice: I. catet-catet II. catet-ipotenuz III. ipotenuz-unghi ascuit IV. catet-unghi ascuit

26

ASEMNAREA TRIUNGHIURILIR Definiie: Trei sau mai multe drepte paralele se numesc echidistante dac sunt situate la distane egale. Teorema paralelelor echidistante: Dac mai multe drepte paralele determin pe o secant segmente congruente, atunci ele determin pe orice alt secant segmente congruente. d1|| d2 || d3 ||.......|| dn i A1A2 A2A3 A3A4 ..............An-1An B1B2 B2B3 B3B4 ................Bn-1Bn

A1

A2

A3

An

B1

B2

B3

Bn

Definiie: Prin raportul a dou segmente, se nelege raportul lungimilor lor, exprimate cu aceeai unitate de msur. Definiie: Segmentele AB, BC, AC sunt proporionale cu segmentele AB, BC, AC dac ntre lungimile lor exist o relaie de proporionalitate de forma:

AB BC AC . = = A' B' B' C ' A' C '

Teorema lui Thales: O paralel dus la una din laturile unui triunghi determin pe celelalte dou laturi sau pe prelungirile lor segmente proporionale.

MN BC

AM AN AM AN MB NC (*) = sau = sau = AB AC MB NC AB AC

27

Teorema paralelelor neechidistante: Mai multe drepte paralele determin pe dou secante segmente proporionale.

BD AB . = DC AC a b ac Dac BC= a, AC=b, AB=c, atunci BD = , CD = , b+c b+c bc AD = [(b + c) 2 a 2 ] - lungimea bisectoarei 2 (b + c)
Teorema bisectoarei: BAD DAC
Teorema reciproc a lui Thales: Dac o dreapt determin pe laturile unui triunghi segmente proporionale, atunci ea este paralel cu cea de a treia latur a triunghiului. Dac

AM AN AM AN MB NC = sau = sau = AB AC MB NC AB AC

atunci

MN BC .
Definiie: Dou triunghiuri se numesc asemenea dac au toate laturile proporionale i unghiurile congruente .

28

A M , B N,C P ABC ~ MNP AB AC BC = = MN MP NP


Teorema fundamental a asemnrii: O paralel la una din laturile unui triunghi formeaz cu celelalte laturi sau cu prelungirile lor un triunghi asemenea cu cel dat. MN BC AMN ~ ABC . Criteriile de asemnare a triunghiurilor.

AM AB = MN AB = MN

si AC MP AC MP

B N ABC ~ MNP (u.u) si A M ABC ~ MNP (l.u.l)

BC ABC ~ MNP (l.l.l) NP

Proprieti ale asemnrii triunghiurilor: 1. ABC ~ ABC .(Reflexivitatea relaiei de asemnare) 2. ABC ~ AMN AMN ~ ABC .(Simetria) ABC ~ AMN , AMN ~ PQR ABC ~ 3. PQR .(Tranzitivitatea)

29

4. Dou triunghiuri isoscele sunt asemenea au o pereche de unghiuri congruente; 5. Dou triunghiuri echilaterale sunt asemenea ntodeauna; 6. Dou triunghiuri dreptunghice sunt asemenea au o pereche de unghiuri ascuite congruente; 7. Dou triunghiuri cu laturile respectiv paralele sunt asemenea; 8. Dou triunghiuri cu laturile respectiv perpendiculare sunt asmenea; 9. Dac dou triunghiuri sunt asemenea atunci raportul de asemnare al laturilor este egal cu: - raportul medianelor; - raportul bisectoarelor; - raportul nlimilor; - raportul razelor cercurilor nscrise; - raportul razelor cercurilor circumscrise; 10. Raportul ariilor a dou triunghiuri asemenea este egal cu ptratul raportului de asemnare al laturilor.

30

RELAII METRICE N TRIUNGIUL DREPTUNGHIC

Formula

distanei
2

dintre
2

punctele

A(x1,y1),

B(x2,y2):

AB= ( x 2 x1 ) + ( y 2 y1 )

Teorema lui Pitagora. ntr-un triunghi dreptunghic, suma ptratelor catetelor este egal cu ptratul ipotenuzei. BC2=AB2+AC2; Teorema reciproc a lui Pitagora: Dac ntr-un triunghi, suma ptratelor a dou laturi este egal cu ptratul celei de-a treia laturi, atunci triunghiul este dreptunghic.

Dac BC2=AB2+AC2 m( A) = 90 0

31

Teorema nlimii: nlimea corespunztoare ipotenuzei este media geometric a proieciilor catetelor pe ipotenuz.

Teorema a II-a a nlimii:

AD 2 = BD DC AB AC AD = . BC

Teorema catetei: Cateta este media geometric dintre ipotenuz i proiecia catetei pe ipotenuz. Teorema unghiului de 300: Cateta opus unghiului de 300 este jumtate din ipotenuz. Dac

AB2 = BC BD ; AC2 = BC CD

Teorema unghiului de 150: nlimea corespunzroare ipotenuzei ntr-un triunghi dreptungic cu un unghi de 150 este un sfert din ipotenuz.

BC ; m(C ) = 30 0 AB = 2

Dac m(C ) = 150 AD =

BC 4

Teorema medianei ntr-un triunghi dreptunghic: Mediana ntr-un triunghi dreptunghic este jumtate din ipotenuz; AM median AM =

BC
2

n orice triunghi dreptunghic are loc relaia: sin2 +cos2=1 Aria triunghiului dreptunghic: AABC=

BC AD AB AC sau 2 2

32

Rapoarte constante n triunghiul dreptunghic:

AB sin C = BC
AC cos C = BC AB tgC = AC AC ctgC = AB

300

450

600

1 2 3 2 3 3 3
1

2 2 2 2

3 2 1 2

3 3 3

Teorema cosinusului: a2=b2+c2-2bccosA

33

Teorema lui Pitagora generalizat:

m(C )90 0 AB2=BC2+AC2+2BC.DC m(C )90 0 AC2=AB2+BC2-2BC.BD m( B)90 0


AB2=BC2+AC2-2BC.DC AC2=AB2+BC2+2BC.BD

m( B)90 0

Distana de la un punct M(x0,y0) la o dreapt de ecuaie: ax+by+c=0;

d (M , d ) =

a x0 + b y 0 + c a2 + b2

34

PATRULATERE DEFINIIE. Se numete patrulater convex patrulaterul n care segmentul determinat de oricare dou puncte ale lui este interior patulaterului. PARALELOGRAMUL Definiie: Se numete paralelogram, patulaterul convex care are laturile opuse paralele. Teoreme: 1. Un patrulater este paralelogram dac i numai dac laturile opuse sunt congruente dou cte dou. 2. Un patrulater este paralelogram dac i numai oricare dou unghiuri opuse sunt congruente i oricare dou unghiuri consecutive sunt suplementare. 3. Un patrulater este paralelogram dac i numai dac diagonalele se njumtesc. 4. Un patrulater este paralelogram dac i numai dac dou laturi opuse sunt paralele i congruente. 5. Un patrulater este paralelogram dac i numai dac oricare dou unghiuri consecutive sunt suplementare.

35

Aria paralelogramului: AABCD= BC AE AABCD=

AC BD sin 2

AABCD= AB BC sin( ABC )


DREPTUNGHIUL Definiie: Se numete dreptunghi, paralelogramul cu un unghi drept. Proprieti: ntr-un dreptunghi, toate unghiurile sunt de 900. ntr-un dreptunghi, diagonalele sunt congruente. Teorem: Dac un paralelogram are diagonalele congruente, atunci el este dreptunghi.

Aria dreptunghiului: AABCD= AB BC

36

ROMBUL Definiie: Se numete romb, paralelogramul care are dou laturi consecutive congruente. Teorem: 1. Un patrulater este romb dac i numai dac are toate laturile congruente. 2. Un paralelogram se numete romb dac i numai dac are diagonalele perpendiculare. 3. Un paralelogram se numete romb dac i numai dac o diagonal este bisectoarea unui unghi.

Aria rombului: AABCD= mic.

Dd 2

, unde D este diagonala mare iar d este diagonala

37

PTRATUL Definiie: Se numete ptrat, paralelogramul care este i romb i dreptunghi. Proprieti: 1. Toate unghiurile ptratului sunt drepte. 2. Diagonalele sunt bisectoarele unghiurilor ptatului. 3. Diagonalele sunt perpendiculare i congruente.

Aria ptratului: AABCD=l2

TRAPEZUL Definiie: Se numete trapez, patrulaterul care are dou laturi opuse paralele, iar celelalte laturi neparalele. Proprieti. 1. Patrulaterul ABCD este trapez isoscel dac i numai dac dac i numai dac

A D si B C .
2. Patrulaterul ABCD este trapez isoscel diagonalele sunt congruente.

38

Aria trapezului: AABCD=

( B + b) h , unde B este baza mare, b este baza mic, iar h 2

este nlimea trapezului. Linia mijlocie: MN= PQ=

Bb 2

B+b ; 2

39

CERCUL 1. Elemente n cerc Definiii: Se numete cerc de centru O i raz r i scriem C(O,r) mulimea tuturor punctelor din plan situate la distana r fa de punctul O.
C (O , r ) =

M OM

= r,r R+

Segmentul care unete dou puncte de pe cerc se numete coard; Coarda care trece prin centrul cercului se numete diametru, iar capetele diametrului se numesc puncte diametral opuse. Poriunea dintr-un cerc determinat de dou puncte distincte ale cercului se numete arc de cerc. Dac extremitile unui arc sunt diametral opuse, atunci arcul se numete semicerc. Se numete interiorul cercului, mulimea punctelor aflate fa de centru la distane mai mici dect raza cercului, iar exteriorul cercului reprezint mulimea punctelor situate fa de centru la distane mai mari dect raza cercului. Mulimea punctelor cercului C(O,r) reunit cu interiorul cercului se numete disc de centru O i raz r: D(o,r). Se numete unghi la centru, unghiul cu vrful n centrul unui cerc.

Msura n grade a unui arc este egal cu msura unghiului la centru corespunztor.

40

m( ADB) = m( AOB)
Msura n grade a unui semicerc este de 1800, iar a unui cerc este de 3600. 1800.................. rad x0.....................y rad y=

x0 1800

Se numete sector de cerc, o poriune dintr-un cerc cuprins ntre dou raze ale sale i arcul pe care l subntind. Se numesc arce congruente , arcele care au aceeai msur. Teoreme: 1. ntr-un cerc, arcelor congruente le corespund coarde congruente i reciproc.

AB CD [ AB] [CD ]

2. ntr-un cerc, diametrul perpendicular pe o coard trece prin mijlocul arcului subntins de coard. 3. Dou coarde ale unui cerc sunt congruente dac i numai dac sunt egal deprtate de centru. [AB ] [CD ] d (O, AB ) = d (O, CD ) . 4. Dac dou coarde ale unui cerc sunt paralele, atunci arcele cuprinse ntre ele sunt congruente.

41

2. Poziiile relative ale unei drepte fa de un cerc.

a) h dreapt secant d (O, h) r b) h dreapt tangent d (O, h) = r c) h dreapt exterioar d (O, h) r Tangente dintr-un punct exterior la un cerc 1. Dintr-un punct exterior unui cerc se pot duce dou tangente i numai dou la cerc; 2. PT PT ' ; 3. PO este bisectoarea unghiului TPT; 4. PO este mediatoarea segmentului TT 5. Msura unui unghi cu vrful pe cerc, care are una din laturi secant i cealalt tangent la cerc, este jumtate din msura arcului cuprins ntre laturile sale.

42

Unghiul nscris n cerc Se numete unghi nscris n cerc, unghiul cu vrful pe cerc ale crui laturi includ dou coarde ale cercului.

m( BC ) ; m( BAC ) = 2 m( BOC ) = m( BC )

m( BD) m( AC ) m( BPD) = 2

m( AC ) + m( BD) m( APC ) = 2

43

3. Poziiile relative ale dou cercuri

a) cercuri secante b) cercuri tangente interioare c) cercuri exterioare d) cercuri tangente exterioare e) cercuri concentrice f) cercuri interioare

r-r<OO<r+r, r>r OO=r-r, r>r OO>r+r OO=r+r au acelai centru OO<r-r, r>r

4.Triunghi nscris n cerc. Patrulater nscris n cerc Definiii: Cercul care conine cele trei vrfuri ale unui triunghi se numete cercul circumscris triunghiului. Centrul cercului circumscris unui triunghi este punctul de intersecie a mediatoarelor triunghiului.

44

Patrulaterul cu vrfurile pe cerc se numete patrulater nscris n cerc(patrulater inscriptibil). Teorem: 1. ntr-un patrulater nscris n cerc, diagonalele formeaz cu laturile opuse perechi de unghiuri congruente. 2. Unghiurile opuse ale unui patrulater nscris n cerc sunt suplementare. Raza cercului circumscris unui triunghi n funcie de aria triunghiului i de laturile sale este:

R=

abc 4S

5. Poligoane regulate nscrise n cerc. Definiie: Un poligon se numete regulat dac este convex, are toate laturile congruente i toate unghiurile congruente. Distana de la centrul poligonului regulat la oricare dintre laturile sale se numete apotema poligonului (a).

45

Perimetrul poligonului regulat cu n laturi este latura poligonului; Aria poligonului regulat cu n laturi este apotema ; Msura unui unghi este Triunghiul echilateral

P = n l unde l este
a P , 2
unde a este

A=

(n 2) 1800 un = 2

l3 = R 3 a3 =

R sau 2

1 a3 = h 3
Aria triunghiului. A3=

l2 3 4

Ptratul

l 4= R 2 ;

a4 =

R 2 L = 2 2
46

Hexagonul regulat

l 6 = R; a6 = R 3 2 3 4

A3 = 6l 2

Un patrulater ABCD este inscriptibil dac unghiul dintre o diagonal a sa i o latur este egal cu unghiul dintre cealalt diagonal i latura opus, sau dac dou unghiuri opuse fac 1800. Un triunghi este circumscris unui cerc sau un cerc este nscris n triunghi dac distanele de la centrul cercului la toate cele trei laturi sunt egale. Raza cercului nscris ntr-un triunghi n funcie de aria triunghiului i de semiperimetrul triunghiului.

r=

S a+b+c , a,b,c sunt laturile triunghiului. unde p = 2 p

47

6. Lungimea cercului. Aria cercului Lungimea cercului: Aria cercului: Lungimea arcului de cerc AMB: l

lc = 2 R
A= R
2

AMB

uR = uR 180 0

Aria sector de cerc AOB:

AAOB =

uR 2 uR 2 = 3600 2

48

Elemente de geometrie n spaiu


Cap I. PUNCTE, DREPTE, PLANE 1. RELAII NTRE PUNCTE, DREPTE I PLANE Noiunile fundamentale ale geometriei sunt:punctul,dreapta, planul, distana i msura unghiurilor. Definiie: Se numete axiom un adevr simplu matematic care nu se demonstreaz deoarece se verific n natur. Axiome: 1.Spaiul este o mulime infinit de puncte. 2. Dreptele i planele sunt submulimi ale spaiului. 3. Orice plan conine cel puin trei puncte necoliniare (nu sunt situate pe aceeai dreapt) 4. Exist patru puncte care nu aparin aceluiai plan (necoplanare). 5. Prin orice dou puncte distincte trece o singur dreapt. 6. Prin orice trei puncte necoliniare trece un singur plan. Definiie: O dreapt este inclus ntr-un plan dac orice punct al dreptei aparine planului. Teorema 1.1 Dac dou puncte distincte ale unei drepte aparin unui plan atunci dreapta este inclus n acel plan. Teorema 1.2 Dac dou plane au un punct comun atunci ele au o dreapt comun. 2. DETERMINAREA PLANULUI Conform axiomei 6 trei puncte necoliniare determin un plan , n plus urmtoarele teoreme ne indic alte situaii de determinare a planului. Teorema 2.1 O dreapt i un punct exterior ei determin un plan. Teorema 2.2 Dou drepte paralele determin un plan. Teorema 2.3 Dou drepte concurente determin un plan.

49

A*
fig 1 fig 2 fig 3

3. CORPURI GEOMETRICE Definiie: Corpurile geometrice se definesc ca fiind mulimea tuturor punctelor, dreptelor i planelor din spaiul cu trei dimensiuni care se gsete n interiorul unei suprafee nchise, inclusiv punctele , dreptele i poriunile de plan care se gsesc pe aceast suprafa. Mulimea tuturor punctelor din spaiu care se gsesc n interiorul suprafeei corpului se numete volumul corpului.
POLIEDRE

Definiie: Corpurile mrginite numai de suprafee plane se numesc poliedre. Poligoanele plane care mrginesc poliedrul se numesc fee(laterale), segmentele comune feelor se numesc muchii i capetele acestor segmente , vrfuri. 4. POZIIILE RELATIVE A DOU DREPTE N SPAIU Definiie: Se numesc drepte coplanare , dreptele care sunt situate n acelai plan. n caz contrar se numesc drepte necoplanare. Dou drepte coplanare pot fi: 1. paralele (nu au nici un punct n comun). 2. concurente au un singur punct n comun). 3. identice (mulimea punctelor lor coincid)

50

Axioma paralelelor: Printr-un punct exterior unei drepte se poate duce cel mult o paralel la dreapta dat. Teorema 4.1: Dou drepte distincte din spaiu, paralele cu o a treia sunt paralele ntre ele.

5. POZIIILE RELATIVE ALE UNEI DREPTE FA DE UN PLAN Definiie: O dreapt este paralel cu un plan dac dreapta i planul nu au puncte comune. Definiie: O dreapt este secant unui plan dac dreapta are un singur punct comun cu planul. Definiie: O dreapt este inclus n plan dac orice punct al ei aparine planului.

Teorema 5.1: O dreapt paralel cu o dreapt din plan este paralel cu planul sau coninut n el.

51

6. POZIIILE RELATIVE A DOU PLANE Planele pot fi de trei feluri: paralele (adic nu au nici un punct comun) , secante( adic au n comun o dreapt dup care se intersecteaz) sau identice( mulimea punctelor lor coincid).

OBSERVATII:

1. O dreapt paralel cu un plan nu este neaprat paralel cu orice dreapt din plan. 2. Dou drepte paralele cu un plan nu sunt neaprat paralele ntre ele. 3. Dou drepte situate n plane paralele nu sunt neaprat paralele. 4. Dou plane, paralele cu o dreapt, nu sunt neaprat paralele ntre ele.

7. TEOREME DE PARALELISM
Teorema 7.1 Dac o dreapt d este paralel cu un plan i coninut ntrun plan care se intersecteaz dup o dreapt g, atunci d i g sunt paralele. Teorema 7.2 Dndu-se dou plane paralele, orice dreapt dintr-un plan este paralel cu al doilea plan. Teorema 7.3 Dac dou plane paralele sunt tiate de un al treilea plan atunci dreptele de intersecie sunt paralele. Urmtoarea teorem stabilete cnd dou plane sunt paralele.

52

Teorema 7.4 Dac un plan conine dou drepte concurente paralele cu un alt plan atunci cele dou plane sunt paralele. Teorema 7.5 ( Tranzitivitatea relaiei de paralelism mtre plane). Dou plane distincte paralele cu un al treilea plan sunt paralele ntre ele. Teorema 7.6 Dou plane paralele determin pe dou segmente paralele pe care le intersecteaz segmente congruente. Teorema 7.7 (Teorema lui Thales n spaiu) Mai multe plane paralele determin pe dou drepte oarecare pa care le intersecteaz segmente respectiv proporionale.

8. UNGHIUL A DOU DREPTE N SPAIU Definiie: Dou unghiuri se numesc suplementare dac suma msurilor lor este de 180 i se numesc complementare dac suma msurilor lor este de 90. Teorema 8.1 Dou unghiuri din acelai plan sau din plane diferite cu laturile respectiv paralele sunt congruente sau suplementare.

Unghiuri congruente respectiv suplementare n acelai plan

Unghiuri congruente respectiv suplementare n plane diferite

53

Definiie: Prin unghiul a dou drepte n spaiu se nelege unghiul de msur mai mic sau cel mult egal cu 90 cu vrful n orice punct al spaiului format prin ducerea de paralele la dreptele date prin acel punct.

Observaii: 1. De obicei, vrful unghiului a dou drepte din spaiu se ia pe una din drepte. 2. Dac dreptele sunt paralele, atunci ele formeaz un unghi de 0. 3. Dac dreptele sunt concurente atunci ele formeaz un unghi plan

9. DREAPTA PERPENDICULAR PE UN PLAN Definiie: Dou drepte din spaiu( concurente sau necoplanare) care formeaz ntre ele un unghi drept se numesc drepte perpendiculare.

a b

54

Definiie: Se numete dreapt perpendicular pe un plan , dreapta care este perpendicular pe orice dreapt din plan. Teorema 9.1 (Criteriul de perpendicularitate.) Dac o dreapt este perpendicular pe dou drepte concurente dintr-un plan atunci ea este perpendicular pe plan. Teorema 9.2 Dintr-un punct exterior se poate duce pe un plan o perpendicular i numai una. Teorema 9.3 Dou drepte distincte perpendiculare pe un acelai plan sunt paralele ntre ele. Teorema 9.4 Printr-un punct se poate duce un plan i numai unul perpendicular pe o dreapt dat. Definiie: Prin distana dintre dou puncte din spaiu se nelege lungimea segmentului determinat de cele dou puncte. Lungimea diagonalei cubului: D= a 3 Lungimea diagonalei paralelipipedului dreptunghic D=

a +b +c

Definiie: Distana dintre un punct i un plan este lungimea segmentului determinat de punct i de plan pe perpendiculara dus din acel punct pe plan.

B 55

Definiie: Distana dintre dou plane paralele este lungimea segmentului determinat de cele dou plane pe o perpendicular comun.

A B

10. SECIUNI PARALELE CU BAZA N POLIEDRE Definiie: A seciona o prism cu un plan paralel cu baza nseamn a intersecta feele laterale cu un plan paralel cu baza. Seciunea rezultat este un poligon asemenea cu cel de la baz chiar mai mult este congruent cu acesta. n urma secionrii se formeaz dou prisme de acelai tip cu prisma iniial. Definiie: Se numete trunchi de piramid corpul geometric obinut prin secionarea unei piramide cu un plan paralel cu baza, situat ntre baz i planul de seciune. Observaie:Clasificarea trunchiurilor de piramid se face dup numrul de laturi ale poligonului de la baz (triunghiulare, patrulatere,hexagonale), dup natura poligonului de la baz(regulat, neregulat) i dup felul cum sunt feele laterale(trapeze isoscele sau nu trunchi drept sau oblic).

56

Definiie: Distana dintre bazele trunchiului se numete nlimea trunchiului. Ea poate fi calculat ca diferena dintre nlimea piramidei din care provine trunchiul i nlimea piramidei noi formate. CORPURI ASEMENEA. RAPORTUL DE ASEMNARE. n urma secionrii unei piramide cu un plan paralel cu baza se obine o piramid de acelai vrf cu baza n planul de seciune asemenea cu piramida iniial. Definiie: Raportul a dou segmente omoloage corespunztor unei perechi de corpuri asemenea se numete raportul de asemnare . Definiie: Raportul ariilor a dou suprafee omoloage corespunztor unei perechi de piramide asemenea este egal cu ptratul raportului de asemnare.

11. SECIUNI AXIALE N CORPURILE CARE ADMIT AX DE SIMETRIE Definiie: Dou puncte Ai B sunt simetrice fa de un punct O, dac O este mijlocul segmentului AB. Definiie: Un punct O este centru de simetrie al unei figuri plane dac orice punct al figurii are simetric fa de O tot un punct al figurii. Definiie: O figur geometric plan admite o ax de simetrie d dac orice punct al figurii are simetric fa de dreapta d tot un punct al figurii. Axa de simetrie a unui corp este dreapta fa de care punctele unui corp sunt simetrice. Axa de simetrie a unei piramide regulate este dreapta ce trece prin vrful piramidei i centrul bazei.

57

Axa de simetrie a unei prisme drepte i a unui trunchi este dreapta ce trece prin centrele bazelor. Definiie: Se numete seciune axial a unui corp, poligonul obinut prin secionarea printr-un plan care conine axa de simetrie a corpului. Observaie: Seciunile axiale n poliedre sunt variabile ca form, iar n corpurile de rotaie sunt congruente. La prisme , planele care conin diagonalele se numesc seciuni diagonale( sau plan diagonal).

Cap II PERPENDICULARITATE N SPAIU 12. Proiecii de puncte, drepte i segmente pe un plan Definiie. Se numete proiecie a unui punct pe un plan, piciorul perpendicularei duse din acel punct pe un plan. Definiie: Se numete proiecie a unei figuri geometrice pe un plan mulimea proieciilor punctelor acelei figuri pe plan.

Teorema 12.1 Proiecia unei drepte pe un plan este o dreapta sau un punct.

58

Teorema12.2 Proiecia unui segment pe un plan este un segment sau un punct.

Teorema 12.3. Proiecia mijlocului unui segment pe un plan sau dreapt este mijlocul proieciei acelui segment pe planul dat sau pe dreapta dat. 13. Unghiul unei drepte cu un plan Definiie: Unghiul unei drepte cu un plan este unghiul fcut de dreapt cu proiecia ei pe plan.

59

Teorema13.1. Unghiul unei drepte cu un plan este cel mai mic dintre unghiurile formate de acea dreapt cu o dreapt oarecare a planului. Teorema 13.2. Lungimea proieciei unui segment pe un plan este egal cu lungimea segmentului nmulit cu cosinusul unghiului format de dreapta suport a segmentului cu planul.

AB= AB.cos u
Teorema 13.3. Aria proieciei unei figuri pe un plan este egal cu aria figurii date nmulit cu cosinusul unghiului fcut de figur cu planul. Aria ABC = Aria ABC cos u

14. Diedru. Unghi plan corespunztor unui diedru. Definiie: Se numete diedru reuniunea a dou semiplane care au aceeai frontier. Definiie: Se numete unghi plan corespunztor unghiului diedru, unghiul format de dou semidrepte coninute n feele diedrului, i perpendiculare pe muchia diedrului n acelai punct. Teorema 14.1. Orice dou unghiuri plane corespunztoare diedrului au aceeai msur.

60

Definiie: Msura unui diedru este msura unui unghi plan corespunztor diedrului. Definiie: Msura unghiului dintre dou plane secante este cea mai mic dintre msurile diedrelor formate de aceste plane i este egal cu msura unghiului format de dou drepte perpendiculare respectiv pe planele date. 15. Plane perpendiculare Definiie: Dou plane se numesc perpendiculare dac unul dintre diedrele determinate de ele are msura de 90. Teorema15.1 Dou plane secante sunt perpendiculare dac i numai dac unul dintre plane conine o dreapt perpendicular pe cellalt plan. Teorema15.2 Dac dou plane sunt perpendiculare, proiecia pe unul dintre plane a oricrui punct din cellalt plan aparine dreptei de intersecie a planelor.

16. Teorema celor trei perpendiculare. Teorema celor trei perpendiculare: Dac o dreapt d este perpendicular pe un plan i prin piciorul ei ducem o perpendicular f pe o dreapt g din acel plan, atunci dreapta determinat de un punct de pe d i de intersecia celor dou drepte din plan, este perpendicular pe dreapta g. Prima reciproc a T. C.3. P: Dac dintr-un punct exterior unui plan ducem perpendiculara pe un plan i perpendiculara pe o dreapt din plan, atunci dreapta ce unete picioarele celor dou perpendiculare este perpendicular pe dreapta dat din plan.

61

A doua reciproc a T.C.3:P: Dac ntr-un punct al unei drepte dintr-un plan se duc dou drepte perpendiculare pe ea, prima exterioar planului i a doua coninut n plan atunci perpendiculara dintr-un punct al primei drepte pe cea de-a doua este perpendicular pe plan. POLIEDRE I CORPURI ROTUNDE
PRISMA TRIUNGHIULAR REGULAT

A l = Pb h At = Al + 2 Ab V = Ab h

PRISMA PATRULATER REGULAT

62

A l = Pb h At = Al + 2 Ab V = Ab h
CUBUL

a= muchia cubului

D = a Al = 4 a At = 6 a V = a3

3
2 2

PARALELIPIPEDUL DREPTUNGHIC

a,b,c = dimensiunile paralelipipedului

D = a2 + b2 + c2 Al = 2 ac + 2bc At = 2 ab + 2bc + 2ca V = a b c

63

TETRAEDRUL REGULAT

l =latura tetraedrului l 6 h= 3 3l 2 3 Al = 4 4l 2 3 At = 4 3 l 2 V = 12

PIRAMIDA REGULAT

A A V

Pb a
l

= A = A
b

2 + A h

64

TRUNCHIUL DE PIRAMID REGULAT

Al =

(P + P ) at B b 2 At = Al + AB + Ab

h V = (AB + Ab + AB Ab ) 3

CORPURI ROTUNDE

CILINDRUL CIRCULAR DREPT

Al = 2RG At = 2RG (G + R ) V = R 2 H

65

CONUL CIRCULAR DREPT

A l = RG At = R (G + R ) V = G
2

R 2 H
3 = H
2

+ R2

TRUNCHIUL DE CON CIRCULAR DREPT

Al = G( R + r ) At = G( R + r ) + R 2 + r 2 V=
2

3 G = h2 + (R r)2

(R2 + r 2 + R r)

66

SFERA

A = 4 R V = 4 R 3

2 3

67

PROBLEME ENUNTURI SI REZOLVARI


CLASA A V A 1. O veveri aduce alune la vizuin n 14 minute, tiind c ea fuge fr alune cu 3m/s i cu alune cu 2m/s, aflai distana de la alun la vizuin. Rezolvare: Timpul ct fuge veveria este dat de raportul dintre lungimea drumului i viteza cu care fuge: Aadar, fie d distana de la alun la vizuin; Atunci d d 5d + = 14 60 = 840 d = 1008m . 3 2 6 2. S se determine a 2007-a zecimal a fraciei Rezolvare:

7 . 11

7 =0,(63); Rezult c a 2007-a zecimal este 6. 11

3. S se afle n N din egalitatea: 2 2 n -4=3(4+4+.42007)

aproape ,rezult 22n=42008

Rezolvare : 2 2 n

=4+ 3 4 + 3 4 2 + ... + 3 4 2007 . Din aproape n


22n=24016 , rezult n=2008.

4. S se determine n ; n<10, astfel nct 9n + 4 2n 1 Rezolvare: Pentru n=1 si n=9 => 9n + 4 2n 1

68

5. S se arate c N=44n+2 .53n+3 .34n + 28n+1 .53n .34n+1 este divizibil cu 2006 n N* Rezolvare: N=(44)n 42 (34)n (53)n 53 + (28)n 2 (53)n (34)n 3 = =28n 34n 53n (24 53+2 3)=
2 (10 3 + 3 )

=28n 34n53n

= 2006

2006

6. S se arate c numrul N=2005 2007 +2006 2008 +2007 2005 +2008 2006 nu e ptrat perfect. u (2005 2007 ) = 5 Rezolvare : Evident u (2006 2008 ) = 6 u (2007 2005 ) = u (7 4*501+1 ) = u (71) =7 u (2008 2006 ) = u (8 2006 ) = 4 u (N) = u (5+6+7+4) = 2 N nu e ptrat perfect.
7. S se afle ultima cifr a numrului

N = 1 2006 +2 2006 +3 2006 ++2005 2006 +2006 2006 Rezolvare : u (1 2006 ) = 1 u (2 2006 ) = 4 u (3 2006 ) = 9 u (4 2006 ) = 6 u (5 2006 ) = 5 u (6 2006 ) = 6 u (7 2006 ) = 9 u (8 2006 ) = 4 u (9 2006 ) = 1 u (10 2006 ) = 0 Ultima cifr a primelor zece numere este 5 u (N) = 5 * 400 + 1 + 4 + 9 + 6 + 5 + 6

69

= 0 u (N) = 1.

31

8. Sa se determine numrul natural a, pentru care: 3 2 27 a 9 a 3 a = 72917 Rezolvare: Relaia data este echivalenta cu:
33a 3 2 a 3 a = (36 )17
3 2

33a + 2 a + a = 3102 3a 3 + 2a 2 + a = 102 Pentru a = 3 327 + 29 + 3 = 102


9. Sa se arate ca numarul: N = 2007 + 2008 ( 1+2+3+.+2006 ) 2 este cubul unui numr natural. Rezolvare: N = 2007 + 2008 ( 2006 2007 ) : 2 2 = = 2007 + 2008 2006 2007 = = 2007 [ 1 + ( 2007+1 ) ( 2007-1 )] = = 2007 ( 1 +20072 1 ) = 20073 10. Sa se compare numerele: A=20062007+20072006 si B=20062006+20072007 Rezolvare: 2006=2005+1 , 2007=2006+1

A=200620062006+20072006=20062006+200520062006+20072006 B=20062006+200720072006=20062006+200620072006+20072006 Cum 2005<2006 si 20062006<20072006 =>A<B

70

11. Se considera numerele a1=4, a2=3a1=4, a3=3a2+42,........ a100=3a99+499,..... a)Sa se determine a2007. b)Sa se compare numerele a100 cu 3150 Rezolvare: Se observa ca pentru orice numr natural n, avem an=3an-1+4n-1=4 => a) a2007=42007 b)a100=4100=2200=(24)50 3150=(33)50 cum 24<33 =>a100<3150 12. S se rezolve ecuaia: x-2004 x-2005 x-2006 x-2007 9 - 159 +79 + 79 = 2007 Rezolvare : Ecuatia dat este echivalent cu:

9x-2007( 7+159-89+9)=2007=> 9x-2007 223=9223=> 9x2007 =91=> x-2007=1=>x=2008


13. Calculai ctul i restul mparirii numrului la 272005 Rezolvare : 972007

D=IC+R R<I 972007 =(7+2)72005 72 =73 . 72005 2005 2005 +2497 =(342+1)7 +49272005 (1712+1)72005 +49272005 = 171272005 +72005 2005 +4927 => =>972007 = (171+49)272005 +72005 Ctul este 220 2005 Restul este 7.

71

14. Gsii toate numerele naturale ptrate perfecte mai mici dect 2007 care la mprirea cu 45 dau restul 36. Rezolvare : Fie n un numr natural. Rezult (conform T impartiri cu rest) c n=45c+36 , n<2007 =>9(5c+4)<2007=>5c+4 223 5c+4 trebuie sa fie patrat perfect =>5c+4 {2 ;3 ;7 ;8 ;12 ;13 ;}cum n=3(5c+4)=>n {6 ;9 ;21 ;24 ;36 ;39} 15. mprind numrul 185 la un numar natural se obine restul 15.Aflai mpritorul. Rezolvare: D=IC+R R<I 185=XC+15=>185-15=XC cum170=2517 si R<X adic X>15 =>X poate fi 17 sau 217=34 sau 517=85 sau 2517=170 Aadar mpritorul aparine mulimi{17,34,85,170}. 16. Sa se arate c nu exist nici un numr natural n astfel nct 20072007 = 7n +2007 Rezolvare : 20072007-2007=7n nu poate fi adevrat pentru nici un numr natural n deoarece ultima cifr a lui (20072007 - 2007)este U(20072007 ) -7=6 ; U(72007)=3 ; 137=6, n timp ce U(7n ) poate fi 7,9,3,sau 1,n nici un caz 6. 17. Dac ab + bc + ac =264, unde a,b,c sunt cifre n baza 10 atunci suma a+b+c este egal cu .; Rezolvare: ab + bc + ac =264 11(a+b+c)=1124 a+b+c=24.

72

18. Aflai numrul natural n pentru care 4n+4n+4n+4n=256 Rezolvare : Este evident ca 4n+4n+4n+4n=4. n n+1 2n+2 8 4 =4 =2 =256=2 . Deci se obtine ca 2n+2=8, adica n=3. 19. Dac ab6 + b8a + 7ab = 2007 atunci ab 17. Rezolvare: 100a+10b+6+100b+80+a+700+10a+b=2007, rezult 111a+111b+786=2007 111(a+b)=1221, a+b=11 Pentru a=3 i b=8 se verific relaia dat. 20. S se arate c numrul n= (20072007-2007)(200620062006)(20052005-2005) este divizibil cu 200. Rezolvare: Ultima cifr a numrului din prima parantez este 6, a celui din a doua parantez este 0, i a celui din a treia parantez este 0, rezult produsul n are ultimele dou cifre 00 i cum 20072007-2007 este divizibil cu 2 rezult c n este divizibil cu 200. 21. S se arate c numrul 1+3+5+7+......+2005+2007 este ptrat perfect. Rezolvare: Notm cu s suma 1+3+5+7+..............+2005+2007 =S i tot cu s suma 2007+2005+2003+.............+3+1=S, dup ce adunm sumele rezult ca 2008+2008+...................+2008 = 2S rezult S=2008.1004:2=10042. 22. S se arate c numrul N= 4 n +1 5 2 n +5 + 2 2 n + 4 25 n + 2 e ptrat perfect, oricare ar fi n natural. Rezolvare: N= 2 2 n + 2 5 2 n +5 + 2 2 n + 4 5 2 n + 4 =

73

2 2 n + 2 5 2 n + 4 (5 + 4) = (2 n +1 5 n + 2 ) 2 3 2 ceea ce nseamn c este ptrat perfect.


23. S se determine valoarea numrului natural a astfel nct numrul N=4a2+2a.a+2a+1 s fie ptrat perfect. Rezolvare: Pentru a= 5 se obine N= 324=182. 24. O persoan mplinete n anul 2007 o vrst egal cu dublul sumei cifrelor anului de natere. Aflai vrsta persoanei i anul cnd s-a nscut. Rezolvare: 19xy+2(1+9+x+y)=2007

1965+2(1+9+6+5)=2007. Aadar x=6 i y=5.


25. S se determine numerele de forma ab tiind c 7 ab + ba = abba : 11 + 1 . Rezolvare:

7(10a+b)+10b+a=(1001a+110b):11+1 71a+17b=91a+10b+1 20a+1=7b a=1; b=3.


26. Suma a patruzeci i dou de numere naturale nenule este 900. Artai c exist printre aceste numere cel puin dou egale. Rezolvare: Dac considerm primele 42 de numere naturale consecutive atunci 1+2+3+4+....42= (42.43):2=903, aadar putem avea n loc de 4 pe 1.

74

27. ntr-o ncpere sunt 11 scaune cu trei respectiv patru picioare. tiind c s-au folosit 40 de picioare metalice pentru scaune, s se spun cte scaune cu trei respectiv patru picioare s-au fcut. Rezolvare: Presupunem c la scaunele care au cte patru picioare le rupem un picior n aa fel nct s avem numai scaune cu trei picioare, n total un numr de 33 de picioare. Aadar din 40 scdem 33 i obinem 7 adic cele 7 picioare rupte provin tot de la attea scaune cu patru picioare. n rest scaunele cu trei picioare sunt n numr de 11-7=4. 28. a) S se arate c numrul N=6+62+63+64+.....+6100 este divizibil cu 7; b) S se afle ultima cifr a lui 2007 2005 N . Rezolvare: a) N=6(1+6)+63(1+6)+........+699(1+6) este divizibil cu 7; b) u(N)=u(20072005)u(N)=u(72005)u(N)=0. 29. Determinai numrul natural x din egalitatea: a) 310 35 : 81 + 3 9 3 : x = 3 25. b) 5 + 125 6 : (5 2 ) 7 5 3 x = 10 + 20 + 30 + ....1000 Rezolvare: a) 3 25 4 : x = 3 25 x = 3 25 4 : 3 25 x = 4. b) 505 x = 10(1 + 2 + 3 + 4 + .... + 100) 505 x = 10 (100 101) : 2 Rezult: x=100.

[(
[

( )] ] )
4

30. S se determine numrul natural n astfel nct 4 n +1 + 4 n = 2 2007 . n +1 n 2 + 3 2 Rezolvare:

75

2 2n+ 2 + 2 2n 5 2 2n = 2 2007 2 n = 2 2007 n = 2007 = 2 2007 2 2n + 3 2n 5 2n


m m +1 N, fracia 2 3 + 2 se 31. S se arate c pentru m, n 7 n +1 8 n + 8 simplific cu 10. Rezolvare: 2 m 3 m +1 + 2 6 m 3 + 2 . Cum ultima cifr a numrtorului i = 7 n +1 8 n + 8 56 n 7 + 8 a numitorului este 0, rezult c fracia se simplific cu 10.

32. S se determine numerele naturale n astfel nct fracia n2 + 3 s se simplifice cu 5. n3 + n2 + 5 Rezolvare:

Fie 5| n+3 si 5| n+ n+5 => 5| (n+ n+5)-(n+3) =>5| n+3 Atunci n poate fi de forma 5K, 5K+1, 5K+2,5K+3, 5K+4 Se observ c numai pentru n = 5k+2 se obine c fracia dat se poate simplifica cu 5 deoarece: (5K+2) +2 = M5+8+2 5.
33. S se determine numrul natural n astfel nct fracia n2 5 N. n3 Rezolvare: n2 5 4 = n +3+ N n 3 4 n {4,5,7}. n3 n3

76

34. ntr-o urn sunt 6 bile albe, 8 bile roii i 12 bile negre. Care este cel mai mic numr de extrageri pe care-l putem face astfel nct s fim siguri c am extras cel puin 5 bile de aceeai culoare. Rezolvare: Presupunand ca extragerile se fac n aa mod nct de fiecare dat extragem o bil de culoare diferit de cea extras anterior(ex. alb, roie, neagr), dup 3 astfel de cicluri de cte 4 bile de culori diferite, deci dup 12 extrageri, cea de-a zecea extragere ne va aduce 5 bile care vor avea aceeai culoare. Deci numrul minim este 13. 35. S se arate c numrul n= 11112222 n baza zece se poate scrie ca un produs de dou numere naturale consecutive. Rezolvare: n=1111*104+2222=1111(104+2)=1111(1041+3)=1111(9999+3)=1111(3.3333+3)=3333(3333+1) =3333*3334 36. S se arate c numrul : N= perfect.

16057 1 1 1 + + ++ este ptrat 2007 1 2 2 3 2006 2007

Rezolvare :

77

1 1 1 1 1 1 + ++ = - + 1 2 2 3 2006 2007 1 2 2 1 1 1 + ... + = 3 2006 2007 1 2007 1 2006 =1 = = 2007 2007 2007 16057 + 2006 18063 = = 9 = 32 2007 2007

N= 3 2
37. S se rezolve ecuaia : xy + x - 4y = 2007

Rezolvare : Ecuaia dat este echivalent cu (x-4) (y+1) = 2003

x-4=1 i y+1=2003 x=5, y=2002 sau x-4=2003 i y+1=1 x=2007, y=0 sau x-4= -1 i y+1= -2003 x=3, y= -2004 sau x-4= -2003 i y+1= -1 x- -1999, y= -2

78

CLASA A VI A ALGEBRA 1. Numerele a,b,c verific simultan condiiile:

1 a) media aritmetic a numerelor a,b,c este 0,4(1) 3 b) a,b,c sunt direct proporionale cu 0,(3), 0,0(3), 0,00(3). S se afle a,b,c. Rezolvare: 37 111 a+b+c= 3 3 = 90 30 a b c a+b+c = = = = 10 3 0, (3) 0,0(3) 0,00(3) 3 3 + + 9 90 900 a=30; b=300; c=3000.
2. S se determine restul mpririi lui 4 Rezolvare:
4 6 = 4 ( 5 +1 ) = 4 ( 5 k +1 ) = ( 4 5 ) k 4 = 1024
n n

6n

la 33.

4 = (1023 + 1) k 4

Rezult c restul mpririi este 4. Observaie: ntodeauna, un numr de forma (a+1)n va fi un multiplu de a la care se adaug 1.
2 2007 x = . 1004 0,5 4 Rezolvare: Se rescrie 41004 ca putere a lui 2, se fac simplificarile si rezulta x=1.

3. Aflai x Q din proporia:

4. S se determine numerele ntregi x,y dac x2+xy-2x-2y=3. Rezolvare:

79

Ecuaia dat este echivalent cu (x+y)(x-2)=3 x+y=1 i x-2=3 x=5, y=-4; x+y=3 i x-2=1 x=3, y=-2; x+y=-1 i x-2=-3 x=-1,y=0; x+y=-3 i x-2=-1 x=1,y=-4.
5. S se rezolve n numere naturale, ecuaia: xy-5=3x+2y-13. Rezolvare: xy-5=3x+2y-13 xy-3x=2y-13+5 x(y-3)=2y2 y 8 2( y 3) 2 2 8 x = = = 2 N, y 3 y 3 y 3 y 3 y 3 D2 y-3=1 y=4; x=0; y-3=-1 y=2; x=4; y-3=2 y=5; x=1; y-3=-2 y=1; x=3.
2 Z astfel nct a 2a + 3 Z . 6. S se determine a a+2 Rezolvare:

a2 2a +3 a(a +2) 4a +3 4a +3 4(a +2) +8+3 = = a+ = a+ = a +2 a +2 a +2 a +2 11 a 4+ Z a +211 a +2 } , a +2 D = {111,1,11 11 a+2=-11 => a=-13 a+2=-1 => a=-3 a+2=1 => a=-1

80

a+2=-11 => a=9.


7. S se rezolve n Z ecuaia: Rezolvare:

x 3 = 2. 3 y

x 3 = 2 xy 9 = 6 y ( x 6) y = 9 3 y x-6=1 i y=9 x=7,y=9 x-6=-1 i y=-9 x=5,y=-9 x-6=9 i y=1 x=15,y=1 x-6=-9 i y=-1 x=-3,y=-1 x-6=3 i y=3 x=9,y=3 x-6=-3 i y=-3 x=3,y=-3

8. Sa se calculeze : 1 1 1 1 + + + ... + 1 2 2 3 3 4 2006 2007 Rezolvare : Suma dat este egal cu 1 1 1 1 1 1 1 1 + + + ... + = 1 2 2 3 3 4 2006 2007 1 2006 =1 = 2007 2007 S-a folosit formula : n +1 n 1 1 1 = . = n(n + 1) n(n + 1) n n + 1 9. S se determine numerele ntregi pozitive a si b care satisfac condiiile a+b=165 si ctul mpririi lui a la b este 10. Rezolvare:

81

a=10b+r , 0 r < b a+b=165=> 10b+r+b=165 => 11b+r= 11 15 => r este divizibil cu 11=> r=11x=> 11(b+x)= 11 15 => b+x=15=> x=1, deoarece r=11x<b => r=11 => b=14 => a=151 . 2x 4 10. S se determine mulimea A = x Z x3 Rezolvare : 2x 4 2 = 2+ Z x 3 2 x 3 D2 = { 2,1,1,2} x3 x 3
x 3 = 2 x = 1

x 3 = 1 x = 2
x 3 =1 x = 4

x 3 = 2 x = 5.
11. S se rezolve n N ecuaia: 4 x + 8 y + 64 z = 192 Rezolvare:

2 2 x + 23 y + 2 6 z = 26 3 : 26 2 2 x 6 + 23 y 6 + 26 z 6 = 3

Cum

2 x 6 < 1, 3y 6 < 1

6z 6 < 1 2x 6 = 0 x = 3 3y 6 = 0 y = 2 6z 6 = 0 z = 1
82

12. S se rezolve n numere ntregi pozitive ecuaia : 3x+5y2=189. Rezolvare: Cum 3x i 189 se divid cu 3 => 5y2 e divizibil cu 3 => y=3a => 3x+5*9a2=189 :3 => x+15 a2=63 => x=3 b => 3 b+15 a2=63 :3 => b+5 a2=21 => b=21, a=0 => x=63, y=0 b=1 , a=2 => x=3, y=6. 13. S se determine numerele ntregi pozitive a si b care satisfac condiiile a+b=165 si ctul mpririi lui a la b este 10. Rezolvare: a=10b+r , 0 r < b a+b=165=> 10b+r+b=165 => 11b+r= 11 15 => r este divizibil cu 11=> r=11x=> 11(b+x)= 11 15 => b+x=15=> x=1, deoarece r=11x<b => r=11 => b=14 => a=151 .

14. a) S se calculeze suma A=1+11+111++ 111 ... 111


2007

b) S se arate c A este divizibil cu 9 c) S se calculeze B=3+33+333++ 33 ... 33


2007

Rezolvare :

1 a) A= (9+99+999++999) 9 10 1 10 2 1 10 3 1 10 2007 1 A= + + + ... + 9 9 9 9

83

1 1 (10 + 102 + ... + 102007 2007) = (111...1110 2007) = 9 9 2007 1 = (111...1109103 ) 9 2003

b) Cum suma cifrelor din care este compus A este 2003 1 +0+9+1+0+3=2016 care este divizibil cu 9 A 9 1 c) B=3A= ( 111...11091033 ) 3 2008

GEOMETRIE 1. Msura unui unghi al unui triunghi isoscel este de 1100. a) Msurile celorlalte dou unghiuri sunt de ; b) Suma msurilor tuturor unghiurilor exterioare triunghiului este de ..; c) Msura unghiului format de bisectoarea unghiului de 1100 cu latura opus este de .; Rezolvare: a) Celelalte dou unghiuri sunt obligatoriu de 350; b) Pentru fiecare unghi al triunghiului exist dou unghiuri exterioare congruente prin urmare suma msurilor tuturor unghiurilor exterioare triunghiului este de 36002= 7200. c) 1800-(550+350)=900.

84

2. n dreptunghiul ABCD cu AB=10 cm i BC =18 cm se ia un punct M pe AB i N pe BC astfel nct AM este de trei ori mai mic dect NC i NC este de dou ori mai mare dect BN. S se afle aria triunghiului MND. Rezolvare:

Din datele problemei, rezult NC=3AM i NC=2BN Cum BC=18, rezult NC=12 i AM=4. Aadar AMND=AABCD-AMBDANCD-AAMD=216-24-72-36=84.
3. n triunghiul ABC se ia D un punct pe latura BC astfel nct AD este congruent cu DC. Dac perimetrul lui ABC este de 46 cm i al lui ABD este de 30 cm se cere lungimea lui AC. Rezolvare: AC= PABC-PABD=46-30=16 cm. 4. n paralelogramul ABCD se ia pe diagonala AC punctele E i F astfel nct AE=EF=FC. S se arate c patrulaterul BEDF este paralelogram i c AABCD=3ABEDF. Rezolvare.

85

Din congruena triunghiurilor ABE i DFC (L.U.L), rezult BE i DF sunt paralele i congruente, rezult BEDF paralelogram. AABCD=6ABEF=3ABEDF 5. Fie ABCD un trapez avnd bazele AD, i BC, cu BC>AD i AD=10cm. Paralela prin A la CD intersecteaz BC n E. tiind c perimetrul triunghiului ABE este 24 s se afle perimetrul trapezului. Rezolvare:

Cum AE este congruent cu DC, rezult c perimetrul trapezului este PABCD= PABE+2AD=24+20=44 cm.

6. Fie ABCD un paralelogram cu AB=8 cm i cu m( )=1500. Se cere msura unghiului A i distana de la B la AD. Dac AD=12 cm, aflai aria paralelogramului.=

Rezolvare:

86

)=1500, rezult c unghiul A are 300 iar BE, unde E este piciorul perpendicularei dus din B pe AD este jumtate din AB. Rezult BE este de 4 cm. AABCD=AD*BE=12*4=48cm2.

Cum m(

7. n rombul ABCD se duc DPAB i BQCD. tiind c msura unghiului A este de 600 s se arate c: a) triunghiul ABD este echilateral; b) patrulaterul BPDQ este dreptunghi; c) s se afle ct la sut reprezint aria patrulaterului BPDQ din aria lui ABCD. Rezolvare: Triunghiul ABD este isoscel cu un unghi de 600, rezult c triunghiul ABD este echilateral. Patrulaterul BPDQ este dreptunghi deoarece are dou laturi opuse paralele i congruente. Cum msura unghiului A este de 600, atunci msura unghiului ADP este de 300 ceeace nseamn c AP este jumtate din AD. Cum AB este congruent cu AD i DPAB chiar n mijlocul lui AB, rezult AABCD=4AAPD. Rezult, ABPDQ=50%AABCD. 8) n patrulaterul ABCD, AC este perpendicular cu BD ca n figura alturat. Dac AC=13cm i BD=8cm s se afle aria lui ABCD.

87

A
Rezolvare:

Cum

AC sunt perpendiculare, BD AC 104 = = 52 cm2 AABCD=AABD+ACBD= 2 2

DB

rezult

9. n figura alturat se dau AC=6,BD=8cm, AE=18,CB=12 cm i A,C,B puncte coliniare. Dac EAAC, BDBC s se afle aria triunghiului ECD.

88

E D

C A B

Rezolvare: AECD=AABDE-AAEC-ADBC=234-54-48=132cm2.
11. Fie ABC un triunghi oarecare. Prin mijlocul lui M al laturii AB se duce paralela la AC care intersecteaz pe BC n N. Prin B se duce paralela la AC care intersecteaz pe AN n D. a) Aflai natura patrulaterului ABDC. b) Dac aria triunghiului AMN=12 cm2 aflai aria patrulaterului ABDC. Rezolvare: MN este linie mijlocie n triunghiul ABD respectiv n BAC, atunci N este mijlocul lui AD i al lui BC, rezult c patrulaterul ABCD este paralelogram. n triunghiul ANM, NM este median, atunci AANM=ANBM, n triunghiul ABD, BN este median, atunci AABN=ABND , n triunghiul ABC, AN este median iar n triunghiul BDC, DN este median, rezult AABDC=4AABN=4*24=96cm2.

89

CLASA A VII A ALGEBRA 1. S se rezolve ecuaia n .

( x 3) + y 2 y = 8
2 2

Rezolvare:

x-3 + y(y-2) =8 => y{0,1,2,3,4} y=0 => x=1; y=1 => x=10; y=2 => x=11; y=3 => x=8; y=4 => x=3.
2. S se determine cifrele consecutive a i b astfel nct
2 2

aa + bb aa bb = 8833. Rezolvare:
Relaia dat este echivalent cu (aa bb ) 2 = 8833 aa bb Cum a i b sunt cifre consecutive, rezult: (aa bb ) 2 = 112 121 = 8833 (10a + a)(10b + b) 11a 11b8833 121 112 a b = 8712 : 121 a b = 72 a=9, b=8 sau a=8, =9.
3. S se determine numerele ntregi n astfel nct n2 3 Z n3 Rezolvare: n 2 3 (n 3) (n 2 + 3n + 9) + 24 24 = = n 2 + 3n + 9 + Z n3 n3 n3 n-3 24 n-3 ={ 1, 2, 3, 4, 6, 8, 12, 24}

90

=> n={-21,-9,-5,-3,0,1,2,3,4,5,6,7,8,9,11,15,27} .
4. S se rezolve ecuaia n N: xy-3x+3y=2016. Rezolvare: Ecuaia dat este echivalent cu (x+3)(y-3)=2007; Cum 2007=32.223 rezult urmtoarele situaii: x+3=9 i y-3=223 rezult: x=6, y=226; x+3=227 i y-3=9 rezult: x=220, y=12; x+3=3 i y-3=669 rezult: x=0, y=672; x+3=669 i y-3=3 rezult: x=666, y=6; x+3=2007 i y-3=1 rezult: x=2004, y=4. 5. S se arate c 1 +

1 2

1 3

+ .... +

1 2007

2007 .

Rezolvare: Cum 1 1 1 1 1 1 , ,........, 2007 2 2007 2006 2007


1+ 1 1 1 1 + + .... + + 2 3 2007 2007 1 1 1 + + + .... + 2002 2007 2007

2007 2007

= 2007 .

6. S se determine numerele ntregi a i b astfel nct

4 6 + 14 = a 2 + b 3; Rezolvare:

91

Ridicm la puterea a doua expresia dat: 2 2 4 6 + 14 = 2a + 2 6ab + 3b ; Din egalarea termenilor asemenea ntre ei rezult : ab=2 i 2a2+3b2=14 rezult: a=1 i b=2.
7. Care numr este mai mare A = 16 15sauB = 18 13 ? Rezolvare: Prin ridicare la puterea a doua se obine: 0<A2<B2 rezult A este mai mic dect B. 8. Dac a Rezolvare:

1 1 =7, s se calculeze a4 + 4 . a a 1 2 ) =49, a obine

Ridicm la puterea a doua relaia dat: ( a a2+ 1 =51 procednd analog a2 1 1 a 4 + 4 = 512 2 a 4 + 4 = 2599 . a a se

9. S se arate c numrul N = 2007 + 2007 2 + 2007 3... + 2007 2007 este divizibil cu 2006 . Rezolvare : 2007 2007 1 2007 2008 2007 Evident N = 2007 = Z 2007 1 2006 fracia se simplic cu 2006 N 2006 . 10. S se rezolve n Z ecuaia :

( x 5 )2

+ y 2 + 2 y =8

Rezolvare :

92

x 5 + y( y + 2) = 8 y = 1 x 5 = 5 x = 10 y = 2 x5 = 0 x = 5
11. S se rezolve n mulimea numerelor ntregi ecuaia 5 x 2 + 6 xy + 5 y 2 = 400 Rezolvare: Ecuaia dat este echivalent cu 4(x+y)2 +(x-y)2=400. Notm x + y = a si

x y = b 4a 2 + b 2 = 400 b = 4b1 4a 2 + 4 2 b1
a 2 + 4b 2 1 = 100 a = 4a1 4a 21 + b 21 = 25 b1 este impar b1 { ,3,5} 1 b1 = 3 a1 = 2 (1) si b1 = 5 a1 = 0 (2 )

= 400

x= y =8

x+ y =0

(1)

x y = 12 x = 10 y = 2

(2)

x y = 20 x = 10 y = 10

12. Aflai X din

X.3 2008 = (3 2008 1) : (1+ Rezolvare:

1 1 1 + 2 + ....... + 2007 ) 3 3 3

1+

1 1 1 3 3 2008 1 + + ........ + 2007 = 2008 , dup formula 3 3 2 3 3 X n +1 1 X 1

1 + X + X + ......... + X n =

93

2 3 2008 2 X 3 2008 = [3 2008 1] 2008 X = 3 3 3 1 2a 3 13. S se calculeze: unde a = 7 11 4 7 3a Rezolvare: 11 + 3 11 3 11 4 7 = = 7 2 a = 2 2 2 2 2 3 3 2 = (2 2 3 )( 3 + 2 ) = 2 6 + 4 3 6 = 6 +1 3 2

14. tiind ca:

a = 3 1 s se calculeze partea ntreag a b a + b numrului a b Rezolvare:

a = 3 + 1 a = 3 + 1, b = 1 b
= = 5+2 3 3+ 2 3 = 9 12

(5 + 2 3 )(3 2 3 ) =

( (

3 +1 +1

) = 3+ 2 3 + 1) 1 3 + 2

3 +1+1 3 +11

15 10 3 + 6 3 12 3 4 3 4 3 3 = = 3 3 3 4 3 3 =1 3

94

15. Se d numarul x = 6 2 5 6 + 2 5 a) S se arate ca x = 4 b) S se calculeze (X+2)2007 Rezolvare:

a. X=

(1 5 ) (1 + 5 ) =

= 1 5 1 + 5 = 1 + 5 1 5 = 2 x = 4

b. x = 2 x + 2 = 0 ( x + 2) 2007 = 0.
16. Daca

66b a . = 2007 , s se calculeze b a 223 9b 66b 66 66 1 = = 223 3 9 660 10

Rezolvare:

a = b 2007

b 2007 223 9b

17. S se calculeze suma S=

2 + 2 2 + 2 3 + .......... + 2 2007 .

Rezolvare:

S=

2 + 22

23 + + 24

2 5 + .......... .. + + .......... .. +

2 2007

)+

+ =

2 1 + 2 + 2 2 + .......... .. + 2 1003 +
2 3 1003

2 2006 =

+ 2 + 2 + 2 + .......... ... + 2

( = [ (2

= 1 + 2 + 2 2 + .......... ... + 2 1003


1004

) 1](

2 + 1 1. 95

)(

+11 = 2 +1 1 =

Am adugat i am sczut 1.
18. Calculai:
Rezolvare:

E = 4 + 2 3 + 7 4 3 + 2 68 351 + 2 68 : 350

4+2 3 = 74 3 =

4+2 42 + = 3 +1 2 2 7 +1 7 1 = 2 3 2 2 <0

(2 ) (3 )
4 17

3 17

E = 3 + 1 + 2 3 + 268 + 351 + 268 : 350 = = 3 + 351 : 350 = 3 + 3 = 6.


19. Determinai n Z astfel nct
Rezolvare:
2 3 5 + 5 1 + 5 1 = = n n 3 5 + 5 1 2 = = Z n { 2,1,1,2} n n 2

14 6 5 + 6 2 5 Z. n

(3 5 )

96

20. Dac x 2 + y 2 = 12 xy , x,y R, x,y>0 x+ y S se calculeze : a ) x y x b) y


c ) partea ntreag a numrului

x . y

Rezolvare : a) ( x + y ) 2 = x 2 + y 2 + 2 xy = 14 xy

( x y ) 2 = x 2 + y 2 2 xy = 10 xy 14 7 35 x+ y = = = x y 5 10 5 b) 5 x + 5 y = 35 x 35 y ( 35 5) x = y (5 + 35 )

x 5 + 35 = y 35 5 2 25 + 35 + 10 35 60 + 10 35 x (5 + 35 ) c) = = = = 10 10 y 35 25
= 6 + 35 x y = 6 + 5 = 11
21. Sa se arate ca a= 1 + 3 + 5 + ...2007 Q

b=

1 3 5 7 ..... 2007 2 tiind ca: 1+3+5++2007= 1004 , => a 97

2 Cum 2003 135..2007 iar 2003 , nu apartine produsului=> b .

22 Determinai x Z pentru care fracia Rezolvare:

x2 +1 Z x+2

x 2 + 1 x ( x + 2) 2( x + 2) + 5 5 = = x2+ Z x+2 x+2 x+2 x + 2 5 x + 2 D 5 = { 1, 5}

x+2=-1 x+2=-5 x+2=1 x+2=5


23. Dac

x=-3 x=-7 x=-1 x=3 .

x 2x + y 1 = 15% atunci = ..............% . y 3x + 2 y + 1 Rezolvare: x 15 3 = 15% = = x = 3; y = 20 y 100 20 2x + y 1 2 3 + 20 1 25 50 = = = = 50% 3 x + 2 y + 1 3 3 + 2 20 + 1 50 100


24. S se rezolve n Z ecuaia: Rezolvare:

x 3 = 2. 3 y

x 3 = 2 xy 9 = 6 y ( x 6) y = 9 3 y x-6=1 i y=9 x=7,y=9 x-6=-1 i y=-9 x=5,y=-9 x-6=9 i y=1 x=15,y=1


98

x-6=-9 i y=-1 x=-3,y=-1 x-6=3 i y=3 x=9,y=3 x-6=-3 i y=-3 x=3,y=-3.


25. S se determine numerele ntregi x,y dac x2+xy-2x-2y=3. Rezolvare: Ecuaia dat este echivalent cu (x+y)(x-2)=3 x+y=1 i x-2=3 x=5, y=-4; x+y=3 i x-2=1 x=3, y=-2; x+y=-1 i x-2=-3 x=-1,y=0; x+y=-3 i x-2=-1 x=1,y=4. 26. S se rezolve n numere naturale, ecuaia: xy-5=3x+2y-13. Rezolvare: xy-5=3x+2y-13 xy-3x=2y-13+5 x(y-3)=2y2 2 y 8 2( y 3) 2 8 x = = = 2 N, y 3 y 3 y 3 y 3 y 3 D2 y-3=1 y=4; x=0; y-3=-1 y=2; x=4; y-3=2 y=5; x=1; y-3=-2 y=1; x=3. 27. Se dau numerele x,y,z, astfel nct x este 20% din y, iar y este 30%
din z. Se cere: a) S se calculeze suma

x y z + + ; y z x z 100 = x 6

b) Dac 50x+10y+z=350 atunci s se afle x, y, z.

Rezolvare: 3 x 20 1 y 30 a) ; = = ; = = y 100 5 z 100 10 x y z 1 3 100 103 . + + = + + = y z x 5 10 6 6

99

b) Cum x =

y 1 3z 3 3 = = z i y = z 50x+10y+z=350 5 5 10 50 10 3z+3z+z=350 7z=350 z=50 x=3 i y=15.

28. Numerele naturale a i b sunt direct proporionale cu 4 i 10. a) Ce procent din numrul a reprezint numrul b? b) Media aritmetic a numerelor a i b este 28. Aflai numerele a i b. Rezolvare: a b 10 250 a) = b= a= a b = 250% din a ; 4 10 4 100 a+b a b 56 28 = 28 a + b = 56 = = = = 4. b) 2 4 10 14 7 29. Fie n , s se afle restul mpririi lui N= 58 n +5 la 25. Rezolvare: N = (5 4 n ) 2 5 5 = [(5 4 n ) 2 1 + 1] 5 5 = = [(5 4 n 1)(5 4 n + 1) + 1] 5 5 = = [(52 n 1)(52 n + 1)(54 n + 1) + 1] 55 Cum (5n-1) 4, (5n+1) 2, (52n+1) 2,(54n+1) 2, rezult c restul mpririi lui N la 25 este restul mpririi lui 55 la 25 care este 21.

100

GEOMETRIE 1. Fie triunghiul ABC cu D AB, E BC astfel nct AADF=8, AAFC=20, Rezolvare: Se tie c n orice triunghi ABC cu E BC exist egalitatea : AABE BE = , atunci AAEC EC vom nota pentru simplitate cu A1=AADF, A2=AAFC, A3=ACEF, A4=ADFE, A5=ABDE;
ACEF=15. Se cere aria patrulaterului BDFE.

A2 AF A1 20 8 = = = A4 = 6, De asemenea A3 FE A4 15 A4 A2 + A3 EC A4 + A3 20 + 15 6 + 15 = = = A5 = 21, A1 + A4 + A5 BE A5 8 + 6 + A5 A5 ABDFE=A4+A5=6+21=27.

2. Un trapez isoscel are lungimea diagonalelor de 5 cm i suma lungimilor bazelor de 6 cm. S se afle aria trapezului.

101

Rezolvare: Fie BM ||AC, M (DC , CM AB, DM=DC+AB=6 i BM AC=5 BDM isoscel cu BE DM, BE nlime

DM BE2=BM2-EM2 BE= 5 2 =4 2 ( AB + CD ) DE DM DE 64 = = ADMB = = 12 . AABCD = 2 2 2

3. n triunghiul dreptunghic ABC, m( A )=900, se ia pe latura BC


punctual F la 2 cm de vrful C i pe AC se ia punctual E astfel nct

CF CA 1 = = . S se calculeze perimetrul triunghiului ABC. CE BC 3 Rezolvare:

Cum FC =2 cm, rezult din relaia dat c CE=32=6 cm, relaie care mai poate fi scris i sub forma : CF CE i cum unghiul C este comun, rezult c triunghiul = CA BC CFE este asemenea cu triunghiul CAB(cazul L.U.L.) CFE este dreptunghic n F. Din Teorema lui Pitagora aplicat n 102

triunghiul CFE FE2=CE2-FC2 FE= 36 4 = 32 = 4 2 . CFE Din ~ CAB CA 1 FE = = AB = 3 FE = 12 2 cm i BC=3CA BC 3 AB Din triunghiul ABC dreptunghic, rezult: BC2=AB2+AC2 (3AC)2=( 12 2 )2+AC2 AC=6 cm i BC= 18 cm PABC=AB+AC+BC= 12 2 +18+6=12( 2 +2) cm.
4. n trapezul dreptunghic ABCD, AB||CD, AB=12 cm, CD=6 cm, CE 1 AD=8 cm se ia pe diagonala AC punctul E astfel nct = . Din E CA 4
se duce o perpendicular pe AC astfel nct ea intersecteaz pe CD n F. Se cere: a) perimetrul trapezului ABCD; b) s se arate c CEF ~ CDA i s se calculeze lungimea segmentului EF; c) s se calculeze distana de la E la AB.

Rezolvare:

103

N C E

Prin punctul E se duce o perpendicular pe bazele trapezului pe care le intersecteaz n MAB i NCD. Cum CD=6 i AB=12 rezult c triunghiul CAB este isoscel (nlimea din C cade chiar la mijlocul lui AB). Din T. lui Pitagora aplicat n ADC CB AC=10. Atunci PABCD= AB+BC+CD+AD=6+8+12+10=36. b) CEF ~ CDA deoarece sunt triunghiuri dreptunghice cu EF CE unghiul C comun, (*) = DA DC CA 5 10 CE 1 Din (*) EF= . = CE = = Cum 4 2 3 CA 4 NE CE NE 1 = = c) Din NE|| AD AD CA 8 4 d(E,AB)=EM=MN-NE=8-2=6 .

5. n triunghiul dreptunghic ABC, m( A )=900, iar AC=12 cm, BC=13


cm. Se cere lungimea bisectoarei BD.

Rezolvare:

104

Aplicnd teorema lui Pitagora n triunghiul ABC, obinem AB=5 cm. Din Teorema bisectoarei, rezult: AD AB 5 AD 5 = = = DC BC 13 AD + DC 5 + 13 AD 5 5 12 10 = AD = = 12 18 18 3 10 5 BD = ( ) 2 + 5 2 = 13 . 3 3
6. n ABC, D AB, F AD, E AC astfel nct
DE BC, FE DC. Se cere lungimea lui AB.

AD 1 = i AF=2cm, DB 4

Rezolvare:

Aplicnd de mai multe ori teorema lui Thales, 1 AD AE AF = = = = 4 DB EC FD 2 FD FD=8 AD=10 10 1 = DB=40 AB DB 4 =AD+DB=50cm.

105

7. n trapezul ABCD, cu BCAD, BC=12 se duce o paralel EF la bazele trapezului, care trece prin intersecia diagonalelor O, astfel nct
A AE 1 = . Se cere AEO . AB 3 A AOD Rezolvare:
A E B O 12 D F

EOBC Din Teorema fundamental a asemnrii AE EO 1 EO = = EO = 4 AB BC 3 12 AE 1 AE 1 = = AB AE 2 EB 2 AD OD EA AD 1 OAD ~ OBC = = = AD = 6 BC OB EB 12 2 A AEO EO 4 2 = = . = AAOD AD 6 3


8. Fie ABC un triunghi i fie AD o bisectoare astfel nct la o treime
fa de vrf, prin punctul P de pe bisectoare se duce o paralel MN la BC. Din N AC se duce o paralel la AB care intersecteaz pe AD in Q si pe BC in R. Dac AB=18, BC=16 si AC=6, aflai perimetrul triunghiului QDR.

Rezolvare:

106

AB BC 18 16 = = 12. Analog DC=4 24 AB + AC rezult: BD =

Cum MN||BC,

AP 1 MP PN 1 MP PN 4 = = = = = MP = 4, PN = . AD 3 BD DC 3 12 4 3 MN=MP+PN = 4 16 16 16 20 4+ = BR = MN = RD = BD BR = 12 = 3 3 3 3 3 Cum PN||RD atunci triunghiurile PQN i DQR sun asemenea: QN PN 4 QN + QR 4 + 20 = = = QR RD 20 QR 20 BM 24 20 BM = QR = QR 20 24 AM AP 1 = = ; AM = 6 BM = 12 QR = 10. AB AD 3 Din formula lungimii bisectoarei, bc 18 6 AD 2 = [(b + c) 2 a 2 ] = [(18 + 6) 2 16 2 = 2 (b + c) (18 + 6) 2

107

60. AD= 60 = 2 15 AP 1 2 15 2 15 4 15 = AP = PD = 2 15 = AD 3 3 3 3 QN PQ PQ + QD 6 PD 6 4 1 = = = = = QR QD 20 5 QD QD 5 5
QD = 5 4 15 10 15 = 36 9

10 15 20 150 + 10 15 + = . 9 3 9 9. S se afle aria triunghiului ABC, dac se cunosc AC=2 cm i m( B )=0,1(6) m( A) i m( C ) = 0,8(3) m( A) ). Rezolvare: 15 1 75 5 = i 0,8(3)= = . Cum 0,1(6)= 90 6 90 6 m( A) + m( B) + m(C ) = 180 0 1 5 m( A) + m( A) + m( A) = 180 0 6 6 m( A) = 90 0 m( B) = 15 0 , m(C ) = 75 0 Aadar, triunghiul ABC este dreptunghic n A . Cum AC se 0 opune unui unghi de 15 ,rezult: BC=4AC=8 i AB= 64 4 = 60 = 2 15. AB AC AABC= = 2 15cm 2 . 2 PQRD=QR+QD+RD= 10+

10. n triunghiul dreptunghic ABC, cu m( A) = 90 0 , AB=6 cm, CD=9


cm unde AD este perpendicular pe BC, se duce prin mijlocul M al lui AC o perpendicular pe AC care intersecteaz pe BC n P, iar AD PM= {N } . Se cere lungimea segmentelor BD, AC, AN, AD.

108

Rezolvare:

Din Teorema nlimii n triunghiul ABC rezult: AD 2 = BD DC = 9 BD, Din Teorema lui Pitagora n triunghiul ABD rezult: AB 2 = AD 2 + BD 2 36 = BD 2 + 9 BD BD = 3 cm AC 2 = BC 2 AB 2 AC = 6 3 . AB AC BC MAN = = AM MN AN Cum ABC ~ 6 6 3 12 = = AN = 6 3 , AD = 3 3 9 AN 3 3 AM MC P este mijlocul lui BC. Aadar: BP=PC=6, MP AB DP=6-3=3.

109

DBA ~

DPN 3 3 3 6 = = DN = 3 3 , NP = 6 , MP = 3, MN = 9 . DP DN NP

11. Fie ABCD un paralelogram cu baza BC=15 cm n care diagonalele BD=18 cm formeaz un unghi de 300 cu AC=10 cm; S se calculeze perimetrul paralelogramului. Rezolvare: Fie AE nlimea paralelogramului dus din A pe BC; AC BD sin 30 0 BC AE = AABCD = 2 2 18 10 = 15 AE AE = 6; 2 Aplicm T. lui Pitagora n triunghiul dreptunghic AEC;

EC = AC 2 AE 2 = 100 36 = 64 = 8; BE=15-8=7; Aplicm t. lui Pitagora n triunghiul dreptunghic ABE: AB = 36 + 49 = 85 P = (30 + 2 85 )cm .
12. Pe ptratul ABCD de latur 12cm se ia punctual M pe BC astfel nct BM =3cm si N pe CD astfel nct DN=1cm. Se cere sinusul unghiului MAN. Rezolvare: Aplicm Teorema lui Pitagora n triunghiurile dreptunghice ABM si AND . . AN= . AM= AABM=18, AMCN=18, AADN=48=>AAMN= AABCD-18-18-48=> AAMN=144-84=60.

110

Pe de alt parte ,AAMN= 10 221 sin MAN = 221

AM AN sin(< MAN ) 2

=>

13. Fie ABCD un trapez oarecare cu bazele AB, CD si O intersectia diagonalelor AC si BD. Paralela prin O la baze taie laturile neparalele in M si N. Demonstrati ca : a) OM = ON; b) Daca PQ este linia mijlocie a trapezului, atunci MN . PQ = AB . CD Rezolvare: a)Deoarece MO AB rezulta ca triunghiurile DOM si DBA sunt asemenea ceea ce conduce la : MO / AB = DO / DB. In mod analog vom avea egalitatea ON / AB = CO / AC. Folosind faptul ca ABCD este trapez, deci triunghiurile AOB si COD sunt asemenea, adica DO / DB = CO / AC, va rezulta ca MO / AB = NO / AB.Deci MO = NO. b)Aceleasi asemanari de triunghiuri ne dau urmatoarele egalitati, in care am folosit faptul ca OM = ON:

MN / AB = 2 MO/ AB = 2 DO / BD si MN / CD = 2 NO / CD = 2 BO / CD, relatii care prin adunare ne dau MN / AB + MN / CD = 2( DO / BD + BO / BD) = 2 BD / BD = 2. 111

Prin impartire la 2 si aducere la numitor comun in relatia de mai sus se obtine MN(AB + CD) / 2 = AB . CD, adica exact ceea ce trebuia demonstrat, pentru ca linia mijlocie PQ este exact semi-suma bazelor.

14.

n triunghiul oarecare ABC se tie: AB=10 BC=14 AC=12. Se cere: a) Aria triungiului b) nlimea AD

Rezolvare: a) A ABC = p( p a)( p b)( p c) unde 10 + 14 + 12 P= =18 2 A= 18(18 10)(18 14)(18 12) =24 6 BC h b) A ABC = 2 14 h 24 6 24 6 = h= 2 7
15. n figura alturat , AOB este un sfert de cerc cu raza de 8 cm i perimetrul suprafeei MPNO este de 20 cm. Dac MP este paralel cu OB i PN paralel cu OA se cere s se afle perimetrul figurii AMNBPA.

112

Rezolvare:

Patrulaterul OMPN este dreptunghi, rezult 2(OM+ON)=20, aadar OM+ON=10. Pe de alt parte diagonalele dreptunghiului sunt congruente, rezult MN=OP=8 cm.(OP este raz). Rezult, AM+BN=AO+OB-(OM+ON)=8+8-10=6. Cum lungimea arcului AB este 2R 2 8 l AB = = = 4 PAMNBPA= AM+MN+NB+lAB= 4 4 6+8+4=14+4.

113

CLASA A VIII -A ALGEBRA 1. Fie f : N R , f (n ) =

1 n +1 + n

S se calculeze: f (1) + f (2 ) + f (3) + ... + f (2007 ) Rezolvare: 1 n +1 n = = n +1 n n +1 n n +1 + n

f (1) + f (2 ) + ... + f (2007 ) = 2 1 + 3 2 + 4 3 + + ... + 2006 2005 + 2007 2006 = 2007 1


2. S se determine x i y din: Rezolvare: Relatia data este echivalenta cu :

x+1=0 x+y-3=0 =>

x=-1; y=4

3. Sa se simplifice fractia: x 4 2 x 3 3x 2 + 8 x 4 E(x)= x 4 + 2x3 x 2 2x

114

Rezolvare : x 4 x 3 x 3 + x 3x 2 + 3x + 4 x 4 E(x)= = x 3 ( x + 2) x( x + 2)

x 3 ( x 1) x( x 2 1) 3 x( x 1) + 4( x 1) x( x + 2)( x 2 1) ( x 1)( x 3 x( x + 1) 3 x + 4) ( x 1)( x 3 x 2 4 x + 4) = = x( x + 2)( x 1)( x + 1) x( x + 2)( x 2 1)


x 2 ( x 1) 4( x 1) ( x 1)( x 2)( x + 2) = = x( x + 2)( x + 1) x( x + 2)( x + 1) ( x 1)( x 2) = . x( x + 1)

4. Fie f: AR, f(x)=x-3; a) S se reprezinte graficul funciei f cnd A=R, A=[0;); A=(-3;2] . b) S se determine mulimea A={n N | n 20, f(n) e ptrat perfect}; f (n + 2) c) Artai c fracia e o fracie ireductibil, f (n + 1) n N ; d) S se rezolve n R inecuaia : f(7-2x) f(3x+2); e) S se determine cel mai mic numr natural x pentru care exist n N astfel nct f(3x+4)=7n; 2x 4 f) S se rezolve ecuaia: f + f ( x + 5) = 1. 3 g) Aflai numerele reale a i b cu proprietatea 2a-3b=4 i c punctual A(a,b) aparine graficului lui f; h) tiind c A(m,n) este punctul de intersecie al graficului lui f cu graficul funciei g:RR, s se determine m i n.

115

i) S se calculeze aria triunghiului determinat de axele de coordonate i graficul lui f. Rezolvare: a)

b) f(n) e ptrat perfect n-3 = k2 n{3,4,7,12,19}. f (n + 2) n 1 c) Presupunem prin absurd c fracia este = f (n + 1) n 2 reductibil adic exist un divizor comun diferit de 1 d = (n 1, n 2) ,
d 1 d n 1, d n 2 d n 1 n + 2 d 1 d = 1 .

Contradicie; rezult c f este ireductibil. d) f(7-2x) f(3x+2) 7-2x-3 3x+2-3 5x 5 x 1 x [1;+) . e) f(3x+4)=7n 3x+4-3=7n 3x+1=7n ; Pentru n=3 x=114 este cel mai mic numr natural pentru care este adevrat relaia dat; 116

f) 2x 4 f + f ( x + 5) = 1. 3 2x 4 3 + x + 5 3 = 1 2 x 4 + 3x = 6 x = 2 ; 3 g) A(a,b) Gf f(a)=b a-3=b a b = 3 (2) b = 2; a = 5 ; 2a 3b = 4 y = x 3 h) x = 7; y = 4 ; A(7,4). y = 2 x + 18 i) Triunghiul format de axele de coordonate i graficul lui f este 33 9 dreptunghic A= = = 4,5 . 2 2
5. a) S se determine funcia f:RR, f(x)=ax+b, a,b R cu proprietatea f(x-2)=3x-2f(2)+3; b) S se reprezinte graficul lui f, c) S se rezolve ecuaia: 3f(x-1) -2f(x)=x+7. Rezolvare: a) Din f(x-2)=3x-2f(2)+3 a(x-2)+b=3x-2(2a+b) +3 (a3)x+2a+3b-3=0 a=3 i b=-1; f(x)=3x-1.

117

b)

y 5

2 O1 -1 2 x

c) 3[3(x-1)-1]-1-2(3x-1)=x+7 x=2.
6. Fie f:RR , g:RR, f(x)= 3 2 x 3 , g(x)= 3 x + 2 2 . a) S se calculeze: f(1)+g(1)+f(-2)+g(-2); b) S se rezolve inecuaia: f(x) g(x); c) S se determine m R astfel nct f( m + 2 )=g( m + 2 3 ). Rezolvare: a) f(1)+g(1)+f(-2)+g(-2)=

2 3 3 .

118

3 2 x 3 3x + 2 2 (3 2 3 ) x 2 2 + 3

b)

2 2+ 3 (3 2 + 3 ) (2 2 + 3 ) x 18 3 3 2 3 3+ 6 x . 3 c) f( m + 2 )=g( m + 2 3 ) x

3 2 (m + 2 ) 3 = 3 (m + 2 3 ) + 2 2 m = m= 3+ 6 . 3

2 2+ 3 3 2 3

7. Dac f:A{0; 3 ;4}, f(x)=3x-2 atunci A=.

2 Rezolvare: f(a)=3a-2=0 a= ; 3 f(a)=4 a=2 2 3+2 A={ , ,2}. 3 3

f(a)=

3 a=

3+2 ; 3

8. Se consider funcia f:RR, f(x)=6x-5. a) Soluia ecuaiei f(x-2)=1 este egal cu...............-; b) Soluiile ecuaiei f(x)=x2 sunt; c) Suma f(1)+f(2)+f(3)+.f(50) este egal cu ..; d) Numrul a R pentru care f(f(a)) =37 este.; e) Soluiile n Z ale ecuaiei f2(x)-8f(x)+7=0 sunt ; f) Lungimea segmentului AB unde A(m,1) i B(2,n) sunt puncte ale graficului lui f este. Rezolvare: a) f(x-2)=1 6(x-2)-5=1 x=3;

119

b) f(x)=x2 x2-6x+5=0 x=1 sau x=5; c) f(1)+f(2)+f(3)+.f(50)= 6(1+2+3+4++50)-550= 3 5051-550=50(153-5)=7400. d) f(f(a)) =37 6(6a-5)-5=37 a=2. e) f2(x)-8f(x)+7=0 f(x)=1 6x-5=1 x=1 sau f(x)=5 6x-5=7 x=2. f) A(m,1) Gf f(m)=1 6m-5=1 m=1; B(2,n) Gf f(2)=n 12-5=n n=7. A(1,1), B(2,7) AB= (2 1) 2 + (7 1) 2 = 37
9. Fie f:RR, cu proprietatea f(xy)=f(x)+f(y). S se calculeze f(2007). Rezolvare: Pentru x=y=0 f(0)=2f(0) f(0)=0; Fie y=0 i x 0 f(0)=f(x)+f(0) f(x)=0, x R f(2007)=0. 10. Trei numere naturale consecutive a,b,c au proprietate ab+bc+ca=146; S se calculeze suma a+b+c. Rezolvare: Fie cele trei numere de forma a, a+1, a+2 Atunci ab+bc+ca=146 a(a+1)+(a+1)(a+2)+(a+2)a=146 2 2 3a +6a+2=146 3a +6a-144=0 a=6. Numerele sunt 6,7,8 iar suma lor este 21. 11. S se determine numerele ntregi a, astfel nct numrul a 2 + 7a + 10 s fie ptratul unui alt numr ntreg. Rezolvare: Z , a.i, a 2 + 7 a + 10 = k 2 4a 2 + 28a + 40 = 4k 2 Fie k 4a 2 + 28a + 49 9 = 4k 2 ( 2a + 7 ) 2 ( 2k ) 2 = 3 2 ( 2a + 7 2k ) ( 2a + 7 + 2 k ) = 9

120

Cum

divizorii

lui

sunt

{ 1,3,9}

2 a + 7 2 k = 1 9 1 9 3 3 2 a + 7 + 2 k = 9 1 9 1 3 3 4a + 14 = 10,10,10,10. 10.6. 6

a = 1,1,6,6,2,5
a { 6,5,2,1} .

12. S se arate c numrul:


116 + 2007 116 2007 2

este raional.

Rezolvare: Notm numrul dat cu k . Prin ridicare la puterea a doua rezult:


116 (116 + 2007 )(116 2007 = k 2 116 116 2 2007 = k 2 116 11449 = k 2 k 2 = 9 k { 3,3}. Aadar, k este numr raional.

121

GEOMETRIE 1. S se arate c suma distanelor de la un punct oarecare din interiorul unui tetraedru regulat la feele sale este egal cu nlimea tetraedrului. Rezolvare : Fie P un punct n interiorul tetraedrului ; => VVABC = VPVAB+VPABC+VPVBC+VPVAC = =1/3.S (d1+d2+d3+d4), unde : - S aria unei fee ; - di distana de la P la una din fee. Cum : VVABC=(S*V0)/3 => => V0= d1+d2+d3+d4 2. Fie VABCD o piramid patrulater regulat avnd muchia bazei de 4 i muchia lateral de 2 5. a) S se calculeze aria seciunii axiale VBD. b) S se afle sin( BVD ). c) S se determine poziia unui punct P, interior diagonalei BD , astfel nct ducnd din acest punct perpendicularele PM respective PN pe dou muchii laterale opuse , s se formeze un segment MN avnd lungimea cea mai mic i s se calculeze aceasta. Rezolvare:

122

BD VO 4 2 2 3 = =4 6 2 2 VB VD sin BVD ABVD = = 2 2 5 2 5 sin BVD b) 2 8 6 2 6 sin BVD = = 20 5 c) Singurul punct pentru care MN este este minim este atunci cnd punctul P coincide cu O, centrul bazei. Se ia P un puct arbitrar pe BD i se duc perpendicularele PM pe VB i PN pe VD. Atunci patrulaterul VMPN este inscriptibil deoarece suma unghiurilor M i N este 1800. Rezult sin BVD = sin MPN (1) i PMN PVN PN (2) sin PMN = sin PVN = VP a) ABVD =

PM MN sin PMN PM PN sin MPN = (3) 2 2 PN = PM PN sin BVD Din (1) i (3) rezult: PM MN VP MN = VP sin BVD MN este minim cnd VP este minim, aadar VP coincide cu VO, nlimea piramidei. 2 6 12 2 MN = 2 3 = . 5 5 APMN =
123

3. Fie VABCD o piramid patrulater regulat avnd muchia lateral VA=4 5 cm i muchia bazei AB=8 cm. Se cere: a) aria total i volumul piramidei; b) msura unghiului format de o muchie lateral cu planul bazei; c) msura unghiului format de o fa lateral cu planul bazei; d) sinusul unghiului format de dou fee laterale opuse; e) Distana de la centrul bazei la o fa lateral; f) lungimea segmentului determinat de centrele de greutate a dou fee laterale opuse; g) sinusul unghiului determinat de dou fee alturate; Rezolvare: a) Fie M mijlocul lui AB; Din Teorema lui Pitagora aplicat n

VAM obinem VM= VA 2 AM 2 = (4 5 ) 2 4 2 = 8 cm; Pb a p 4 AB VM Al= = = 128cm 2 . 2 2 At=Al+Ab=128+64=192 cm2 VO este nlime n triunghiul echilateral VMN VO l 3 8 3 = = 4 3 cm; = 2 2 A h 64 VO 64 4 3 256 3 3 V= b = = = cm ; 3 3 3 3

124

b) VA formeaz cu (ABC) AC 8 2 = = =4 2 2 2 VO 4 3 6 tg (VAO) = ; = = AO 4 2 2

unghiul VAO .

Cum AO

c) Cum VMN este echilateral m(VNO) = 60 0 ; d) Unghiul format de planele VAB i VDC este unghiul determinat de nlimile celor dou triunghiuri i are msura de 3 ; 600, sin 600= 2 e) Vom calcula distana de la centrul bazei, O, la planul VDC. Fie P VN astfel nct OP VN (1) Evident c CD (VON) OP CD (2)

125

i (2) OP (VDC) VO ON 4 3 4 d(O,(VDC))=OP = = = 2 3 cm; VN 8 f) Unind centrele de greutate a dou fee opuse se formeaz dou triunghiuri asemenea n care raportul de asemnare este 2 . Fie G1G2 segmentul determinat de centrele de egal cu 3 greutate G1G2 2 16 = G1G2 = cm. MN 3 3 g) Fie AR VD i CR VD. Cum triunghiurile VAD i VDC sunt isoscele AR i CR sunt congruente triunghiul ARC este isoscel; Cum VD (ARC) i OR (ARC) OR VD OR este nlime n triunghiul ARC; VO OD 4 3 4 2 4 30 = = n VOD dreptunghic, OR = VD 5 4 5 Din (1) AC OR 8 2 4 30 32 15 = ; Pe de alt parte, = 2 25 5 AR RC sin R (*) AARC= 2 Mai rmne s aflm lungimea segmentelor AR i RC care se afl din triunghiul isoscel VDC, scriind aria sa n dou moduri: CD VN VD CR 8 8 4 5 CR 16 5 AVDC = = = CR = = AR ; 2 2 2 2 5 AARC = Din relaia (*) Observaii: 126 32 15 (16 5 ) 2 sin R 15 = sin R = ; 2 5 4 5 2

O alt cerin a problemei putea fi determinarea msurii unghiului dintre dreptele AC i VD care se determin uor din punctual g) unde s-a artat c VD (ARC) VD AC, rezultnd c dreptele fac un unghi de 900 ntre ele; Deesemenea se poate cere determinarea unui punct Q i VQ , astfel nct aria triunghiului AQC s a raportului QD fie minim, aceasta avnd loc atunci cnd Q coincide cu R, deoarece atunci distana OQ =OR este minim; VQ=VR se afl din triunghiul dreptunghic VOR.
4. n prisma patrulater regulat ABCDABCD, cu muchia bazei de 6 cm, se ia M respectiv N mijloacele muchiilor CC respectiv DD. tiind c dreptele BM i CN formeaz ntre ele un unghi de 600, s se afle muchia lateral. Rezolvare:

CN|| MD triunghiul MBD este isoscel cu msura unghiului M de 600, rezult c este chiar echilateral.

BD= 6 2 =MB=MD, BCM MC=6.

Din

triunghiul

dreptunghic 127

Aadar,

lungimea

muchiei

laterale

este

CC=12.

5. n cubul ABCDABCD, cu muchia de 4 cm se cere distana de la punctul A la planul ABC. Rezolvare: Se duce AP perpendicular pe AB. Evident din BC AB, BC A' B ( A' AB) ( A' BC ) AP ( A' BC ) AA' AB =2 2. d(A, (ABC))=AP= A' B

6. Un paralelipiped dreptunghic are dimensiunile direct proporionale cu numerele 2 , 8 ,3 i volumul de 648 2 . S se afle lungimea diagonalelor paralelipipedului. Rezolvare:

Fie a,b,c dimensiunile paralelipipedului, atunci: a b c = = = k a= 2 k , b= 8 k, c=3k. 2 8 3 V=abc; 648 2 =12k3 k= 3 2 a=6, b=12, c= 9 2 . Atunci lungimea diagonalei este: 128

D = a 2 + b 2 + c 2 = 3 38 .

7. n cubul ABCDABCD cu muchi de 8 cm se ia M mijlocul lui CC. S se determine cosinusul unghiului format de planele ABM i ABC. Rezolvare:

Se va aplica formula ariei proieciei unui triunghi pe un plan: S=S.cos (*),unde S este aria proieciei, S este aria triunghiului care se proiecteaz. AABC=AABMcos AB=8 2 , AABC=32, Se duce o paralel prim M la BC care este perpendicular pe BB n punctul N(N este mijlocul lui BB), iar din N se duce o perpendicular pe AB n P. Conform teoremei celor trei perpendiculare rezult c MP este perpendicular pe AB. Aadar triunghiul BPN este asemenea cu BBA. 129

Rezult PN=2 2 . Cum MN este 8, atunci cu T lui Pitagora obinem: MP2=MN2+PN2 A' B MP 8 2 6 2 = = 48 Din relaia (*) MP=6 2 . AABM= 2 2 rezult 32 2 cos= = . 48 3
8. n paralelipipedul dreptunghic ABCDABCD se cunosc muchiile bazei AB=2 5 si BC=4 5 , iar aria sa total este 70. Se cere tangenta unghiului fcut de AO cu planul BDB unde O este centrul bazei de sus.

Rezolvare: A t =2(ab+ac+bc) Din A t =70 nlimea paralelipipedului este 5 . Ducem AE BD, E BD, AE nlime n ABD, AE=4, Ocentrul bazei ABCD AEO dreptunghic n E AE 4 = ' tg(OA,(BDB))=tg(A O E) EO' 3

130

PROBLEME PROPUSE Operaii cu numere ntregi


1)(5 23) ( 11 + 13 + 2 14 ); 2 )36 : ( 2 ) + 4 ( 9 ) 45 : ( 15) + 3 ( 10 ) : ( 15);
5

3)5 ( 6) 4 ( 2) + 6 (+ 7 ) 3 ( 15);

4 )5 3 58 : ( 25) 2 5 35 : ( 6 ) + (16 ) : ( 8) ; 5)24 : {[30 + ( 6 ) ( 5)] 60 + 2} + 15 : [( 3) ( 1)];


5 3 3

( 8)(11

6 )( 4 ) ( 8) 16 4 : 8 5 ;
2 4 2 4 60 2 2 2

7 ) 5 30 5 20
2007

) ( 5) ( 5) ( 5) ( 5) 1)(16 1) : (2 3 5) ;
40 30

]( )
{

70

9 )( 2 )

10 ) 2 5 (3 2 ) : 2 2 + 2 6 2 2 5 ; 11)1 + 2 + 3 + 4 + ... + 2006 + 2007; 12 )2 + 4 + 6 + ... + 2004 + 2006; 13)1 + 3 + 5 + 7 + ... + 2005 + 2007;
2 2

{ [

: 2 2005 10 2 2 ( 4 ) : 4 4 2 ;
5

]} (

]}

14 )2 + 2 2 + 2 3 + ... + 2 2006 + 2 2007 ; 15) 2 2007 2 2006 2 2005 : ( 8) ;


668

17 )2 2007 2 2006 + 2 2005 + ... + 2 + 1 ; 4 2 4 ; 19 )( 3) { 5 + 3 [( 6 ) (+ 2 ) + ( 5) ( 6) + ( 2)] ( 3)};


3 9 4 8 4 2

( 18)(9 3 ) (8

16 ) 2 333 + 2 333 3 222 : ( 9 ) ;


111

)(

20 ) 12 + 12 { 13 [ 5 ( 2 ) ( 3)] 6}; 13

131

21)( 3) 22)( 2)

11 12

100

23){( 18) : ( 6 ) ( 2) ( 3) [20 + ( 4) ( 5)]} ( 57 ); 24)( 2) ( 2) + ( 2)


46 45 0 1

] : [(3 ) ( 3) ] ( 4) ( 8) : ( 2) ; :[ 16 : 2 : ( 10 + 8) 2 ( 2) ];
12 10 9 12 7 2 25 4 26 8 62 44 2

25)( 1) 1 + ( 1) 2 + ( 1) 3 + ( 1) 4 + ... + ( 1)
3

[ [

]: [7 ( 2) ];
44

2005

2006 +

+ ( 1)

2006

2007;

26)( 2) 2 2 2 1 : 2 2 + ( 2) + 2 1 ;
2 2

27 )10 11 10 + 11 10 11 10 + ... + 11 10 11 10 + 11 1;
10 9 8 7 2

) ( (

)] [

28)2007 2 2007 2 2 2007 2 2006 2 2005 ... 21 2 0 ; 29)5


2008

2007

2006

19 25

1003

30)S se compare numerele: b)7100 cu 7200 ; a)36 cu 32 ; e)248 cu 332 ; d)330 cu 245 ; 107 46 45 g)2 cu 5 -5 ; h)(-3)75 cu (-2)125; 2 j)(-2)40 cu (-16)10; k)(54 )5 cu 5 2 ;

c)1023 cu 523 ; f)251 cu 334 ; i)25100 cu 12549; l)(-2)71 cu (-3)71.

132

Mulimea numerelor raionale 0 1 2007 1) 0, (7 ) : + 2, (3) + ; 49 2006


2 1 1 2 2) 1, (3) (0,5) : 3 + 0, (6) : ; 3 2

3) (9 : 0,15 + 1 : 0,2) : 0,01; 4 ) 0,0(6 ) + 5) 6) 15 1,28 2[0,125 0,08(3)]; 64


3

1 7 2 2 10 (0,01) : (0,7 ) + 2; 100 7 10

[0,4 3, (3) 1] : 0, (5) + 0,1;

2 1 4 3 1 3 : 3 + : 3 9 9 31 4 7 7) ; 4 1 3 3 6 :1 3 : 5 2 5 8 8)[1, (6) + 0,25] : 1,91(6 )


10 3 5 22 3 11 7 3 9) 2 : 2,75 4 1 + 1 ; 4 4 8 4 2 7 4 10 ) + + ; 3 5 15 3 13 1 11) + + ; 7 14 2

12 ) 23,56 + 5 63,64 0,56 + 0,04;

133

2 1 14 2 1, (6) + ; 3 2 4 2 14) 2,5 6,7 + 2, (3) + + 3,2; 15 2 7 3 2 15) 2 + + + ; 3 12 4 6 13) 2 15 3 16) 3 8 ; 5 18 7 5 18 38 17 ) ; 36 19 45 25 18) ( 0,2 ) ( 1,4 ) ; 49 1 17 2 2 36 19 ) ; 19 51 1 81 40 3 5 6 20) : : ; 16 8 4

134

9 21)( 4,5) : ( 0,7 ) : ; 14 3 5 3 26 22 ) : ; 4 4 1 1 23) ; 49 7 5 5 5 24 ) : ; 7 7 7


3 5 2 16 2 8 25) : ; 81 3 27
83 14 95 14 15 6

9 19 3 20 27 12 27 89 26 ) : ; : 25 5 125 125 3 2 2 5 7 27 ) 2 ; 3 3 6 12 4 1 1 1 + 2 3 4 28) ; 5 25 : 6 26 1 1 15 7 11 29 ) 2 + [ 3,2 (4,8 5)] + 0,2. 3 2 11 22 13

135

RAPOARTE I PROPORII

1. Dac

x 3 3x y = atunci = y 5 x+ y

2. Numerele a,b,c sunt invers proporionale cu numerele a+b = 0,(3);1/5 respectiv 52. Atunci bc 3. Suma a trei numere este 680. Dac se mrete primul cu 50% din el, al doilea se micoreaz cu 25% din el,iar al treilea se micoreaz cu 5, numerele devin egale.Ct la sut reprezint primul numr din al treilea? 4. Dac

a2 b+3 , atunci suma a2+b2 este egala cu = 3b 2+ a

5. Artai c 212+210+20 este ptrat perfect. 6. Fie a,b,c,dN,b>c. tiind c (a;b)(c;d)=(2;4) i (a;b) (c;d)=(0,6) atunci media aritmetic a numerelor a,b,c,d este 7. Dac

x yx = 20% atunci = ........ y 3x + y

a b b c = i = i a2+b2+c2=16500 atunci media 2 5 10 7 aritmetic a numerelor a,b,c este


8. Dac

136

9. Dac A={xN*| 22x+3<612} atunci probabilitatea ca un numr din A s fie par este

10. Dac

5x + 7 y x = 60% atunci = y 9x 5 y

11. Numerele pozitive x,y,z sunt invers proporionale cu 1 1 1 ; ; .Ct la sut reprezint numrul cel mai mic din 3 4 5 numrul cel mai mare? 12 .Dac

x+ y x = 5 atunci = yx y

99 98 2100 299 2 2 = 13. 3 x

=> x=

14. Dac 30% din lungimea unui drum reprezint 12 km,atunci drumul are o lungime dekm. 15. Dac

x y = 5 3

=>y reprezint %din x

x reprezint % din y Dac x=85 =>y=


16. Dac

a b b c = si = , a+b+c=210,atunci a= b= 2 5 10 7

c= 137

a b b c = si = si a2+b2+c2=16500,atunci media 2 5 10 7 aritmetic a numerelor a,b,c este


17. Dac 18. Numerele a,b,c sunt invers proporionale cu numerele 0,2; 1 0,(3); . tiind c suma numerelor e 160 s se afle: 2 a) a,b,c; b) ct la sut reprezint a din c. 19.Suma a doua numere este 120. tiind ca unul din ele este cu 25% din cellalt, s se afle: a)cele doua numere; b)raportul dintre cel mai mic si cel mai mare . x2 x 2 3x + 4 y 20. = => = ; 2 = y 5 y y

21.

x 0,3 x+ y => = = ; y 0,5 y

x2 y2 = ... y2

22.

x x = 5%, atunci = y 5x + y

23. Dac x,y,z sunt direct proporionale cu 0,5; 0,(3); 0,(6) i 2x-3y+4z=12, atunci x-y+z= 23. 30% din

2 5x = 4 => x= 3 + 2x

138

ISTORICUL NOIUNILOR MATEMATICE

Sec. 18 .e.n. mesopotamienii creeaz primele tabele de nmulire; sec. 6 .e.n. este cunoscut asemnarea triunghiurilor de ctre Thales; Sec. 5 .e.n. pitagorienii introduc noiunile de numr prim, numr compus, numere relativ prime, numere prime perfecte; Sec. 4 .e.n. Aristotel (384-322 .e.n) filozof grec a introdus noiunile de perimetru, teorem, silogism. Sec. 3 .e.n. Matematicianul grec Euclid(330-275 .e.n ) cel care a ntemeiat celebra coal din Alexandria (n 323 .e.n) a introdus noiunile de semidreapt, tangent la o curb, puterea unui punct fa de un cerc sau sfer, sau denumirile de paralelogram, poliedru, prism, tetraedru. A enunat teorema catetei i a nlimii pentru un triunghi dreptunghic i a demonstrat concurena mediatoarelor unui triunghi; n prima carte din Elementele lui Euclid este cunoscut teorema mpririi cu rest i algoritmul lui Euclid pentru aflarea c.m.m.d.c. a dou numere ntregi Apolonius din Perga(262-200 .e.n), unul din cei mai mari geometri ai antichitii introduce pentru prima dat denumirile pentru conice, de elips, hiperbol, parabol i noiunile de focare, normale i definete omotetia i

139

inversiunea i d o aproximare exact a lui cu patru zecimale. este dat aria triunghiului n funcie de laturi sau n funcie de raza cercului nscris i semiperimetru; Eratostene din Cyrene(275-195 .e.n) introduce metoda de determinare a tuturor numerelor prime mai mici dect un numr dat, metod cunoscut sub numele de Ciurul lui Eratostene 85-168 matematicianul grec Ptolemeu prezint n cartea sa Almagest, pe lng vaste cunotine de astronomie i trigonometrie i diviziunea cercului n 360 de pri congruente i exprimarea acestora n fracii sexagesimale. Sec. 3 s-a dat formularea teoremei celor trei perpendiculare de ctre Pappos; acesta a mai dat i definiia conicelor precum i teorema despre volumul corpurilor de rotaie Sec. 7 sunt cunoscute regulile de trei direct i invers de ctre Bragmagupta, matematician indian; Arhimede(287-212 .e.n) precursor al calculului integral, a determinat aria i volumul elipsoidului de rotaie i ale hiperboloidului de rotaie cu pnze. 1202- Leonardo Fibonacci (1170-1240) matematician italian introduce notaia pentru fracia ordinar; 1228- Fibonacci introduce denumirea pentru numrul zero, precum i sistemul de numeraie zecimal. Tot prin opera sa Liber abaci sunt introduse pentru dat n Europa numerele negative, fiind interpretate ca datorii; 1150- este descris extragerea rdcinii ptrate i a celei cubice n cartea Lilavati a matematicianului indian Bhaskara(1114-1185), tot el prezint i operaiile de nmulire i mprire cu numere negative;

140

1515- rezolvarea ecuaiilor de gradul al treilea cu o necunoscut de ctre Scipio del Fero, iar mai trziu de Niccolo Tartaglia n 1530, i pe acelea de gradul al patrulea de Ludovico Ferrari n 1545. Acestea au fost fcute cunoscute abia n 1545 de ctre Girolamo Cardano(1502-1576) n lucrrile sale, dei promisese autorilor lor s nu le divulge; 1591-matematicianul francez Francois Viete(15401603) introduce formulele cunoscute sub numele de relaiile lui Viete; 1614- inventarea logaritmilor naturali de ctre John Neper(1550-1617); 1637- este introdus noiunea de variabil de ctre Rene Descartes(1596-1650), cel care a introdus literele alfabetului latin pentru notaii i a folosit coordonatele carteziene (definite dup numele su), reducnd problemele de geometrie la probleme de algebr; 1640- este introdus denumirea pentru cicloid de ctre Galileo Galilei (1564-1642); 1654- nceputul crerii teoriei probabilitilor datorat corespondenei dintre Pierre Fermat(1601-1665) i Blaise Pascal(1623-1662) i dezvoltarea combinatoricii odat cu apariia lucrrii lui Pascal, Combinaiones; 1656- matematicianul englez John Wallis(1616-1703) 1 1 = , = 0 i a introduce simbolul cu notaiile 0 denumirilor de interpolare respectiv mantis 1670- este determinat semnul sinusului i desenat sinusoida respectiv secantoida de ctre John Wallis); 1678- este dat teorema lui Ceva de ctre Ceva Giovani(1648-1734); 1679- n Varia opera mathematica aprut postum, a lui Pierre Fermat(1601-1665), a fost dat Marea

141

teorem a lui Fermat, reguli de integrare, definiia derivatei. 1692- este scris primul manual de calcul integral de ctre matematicianul elveian Jean Bernoulli(16671748) Lectiones mathematicae de methodo integralium aliisque, tiprit abia n 1742 i de asemenea a mai scris un manual de calcul diferenial, descoperit abia n 1920. Regula lui lHospital este dat de ctre Jean Bernoulli lui Guillaume de lHospital pe care acesta o public n 1696; 1690- este propus denumirea de integral de ctre Jacques Bernoulli(1654-1705) 1692- sunt descoperite proprietile spiralei logaritmice (Jacques Bernoulli) 1694- este descoperit curba numit lemniscat, caracterizat de inegalitatea (1+x)n 1+nx (Jacques Bernoulli); 1696-1697- introducerea calculului variaional, punerea problemei izoperimetrelor de ctre Jean Bernoulli. 1705- este dat Legea numerelor mari de ctre Jacques Bernoulli; 1711- realizarea dezvoltrii n serie a funciilor ex, sinx, cosx, arcsinx, de ctre matematicianul englez Isaac Newton(1642-1727) cel care a pus bazele calculului diferenial i integral concomitent cu Gottfried Leibniz(1646-1716); 1729- este demonstrat existena rdcinilor complexe n numr par a unei ecuaii algebrice cu coeficieni reali de ctre Mac Laurin Colin(1698-1746; 1731- utilizarea sistemului de axe perpendiculare pentru a determina poziia unui obiect n funcie de cele trei coordonate;

142

1733- crearea trigonometriei sferoidale de ctre Alexis Clairaut(1713-1765); 1735- Matematicianul elveian Leonhard Euler(17071783) introduce i calculeaz constanta 1 1 1 e= lim(1 + + + ... + ln n) =0,577215..., n; n 2 3 1739- introducerea conceptului de integral curbilinie de ctre Alexis Clairaut; 1746- relaia lui Stewart este demonstrat de Mathew Stewart dup ce n prealabil ea i fusese comunicat de ctre Robert Simson n 1735; 1747 este enunat problema celor trei corpuri de ctre Clairaut; introducerea metodei multiplicatorilor nedeterminai n studiul sistemelor de ecuaii difereniale de ctre Jean Le Rond DAlembert(1717-1783); 1750- Gabriel Cramer d o regul de rezolvare a sistemelor cunoscut sub denumirea de metoda lui Cramer; 1755- sunt puse bazele calculului variaional de ctre Lagrange(1736-1813) concomitent cu Euler, 1765- nceputul crerii geometriei descriptive de ctre Gaspard Monge(1746-1818); 1766- crearea mecanicii analitice de ctre Joseph Lagrange(1736-1813) cu enunarea principiului vitezelor virtuale i a ecuaiilor Lagrange; 1767- demonstrarea iraionalitii lui de ctre Heinrich Lambert(1728-1777); 1768- demonstrarea existena factorului integrant la ecuaiile difereniale de ordinul nti de ctre DAlembert;

143

1771- a fost dat ecuaia planului normal i formula distanei dintre dou puncte din spaiu de ctre matematicianul francez G. Monge; 1775- introducerea noiunilor de soluie general i soluie particular n teoria ecuaiilor difereniale de ctre Leonhard Euler; acesta a introdus i funcia (n) - indicatorul lui Euler, precum i notaiile e, i, f(x)i a creat teoria fraciilor continue; 1780- au fost introduse liniile de curbur ale suprafeelor(G. Monge); sunt descoperite funciile automorfe de matematicianul francez Henri Poincare(1854-1912); 1785- a fost dat ecuaia planului tangent(G. Monge); 1796- este dat Teorema lui Fourier de determinare a numrului rdcinilor reale cuprinse ntr-un interval, de ctre Joseph Fourier(1768-183); 1797- este dat formula creterilor finite, cunoscut sub denumirea de teorema lui Lagrange; 1798- au fost considerate cosinusurile directoare ale unei drepte(G. Monge); este introdus simbolul [.], pentru partea ntreag de ctre Adrien Marie Legendre(1752-1833) (1752-1833); 1807-, 1822 sunt date seriile Fourier care au contribuit la crearea teoriei analitice a cldurii. 1812- este introdus seria hipergeometric de ctre Carl Friedrich Gauss(1777-1855) matematician german, cel care a demonstrat teorema fundamental a algebrei; 1816-1835- Augustin Cauchy(1789-1857), fondatorul analizei matematice moderne, a enunat criteriul de convergen al seriilor, criteriu care-i poart numele, a dat primele teoreme de existen din teoria ecuaiilor difereniale i al ecuaiilor cu derivate pariale, a

144

introdus noiunile de afix, modul al unui numr complex, numere conjugate, transpoziie; 1820- introducerea noiunii de raport anarmonic de Chasles Michel(1793-1880), fondatorul ctre geometriei proiective alturi de matematicianul francez Jean Poncelet; 1822 introducerea funciilor Bessel de ctre Friedrich Bessel; este introdus notaia pentru integrala definit

f ( x)dx , de ctre Fourier.;


a

este propus denumirea de reprezentare conform de ctre Gauss; cercul lui Euler sau cercul celor nou puncte este considerat pentru prima dat de ctre Charles Brianchon , Jean Poncelet i Karl Feuerbach, atribuinduse din greeal numele lui Euler acestei teoreme; 1823-1831- nceputul crerii primei geometrii Bolyai(1802-1860) neeuclidiene de ctre Jano concomitent i independent de cea a lui Lobacevski. 1824este dat denumirea de geometrie neeuclidian de ctre Gauss; Niels Abel(1802-1829) demonstreaz imposibilitatea rezolvrii cu ajutorul radicalilor, a ecuaiilor algebrice de grad mai mare dect patru; 1825- Abel introduce integralele ce-i poart numele; 1827- este creat teoria funciilor eliptice de ctre Abel; 1828 sunt introduse formele fundamentale ale suprafeelor i curburii total a unei suprafee(curbura Gauss) de ctre Gauss; 145

demonstrarea teoremei lui Fermat pentru n=5 de ctre Dirichlet (1805-1859); matematicianul german 1830- este propus denumirea de grup cu nelesul actual de ctre matematicianul francez Evariste Galois(1811-1832); 1831- definitivarea calculului cu numere complexe de ctre Gauss ; 1834- introducerea noiunii de factor de discontinuitate, referitor la integralele 1837- introducerea notaiilor pentru limite laterale de ctre Dirichlet i a funciei care i poart numele, funcia Dirichlet; W. Hamilton introduce termenul de asociativitate a unei legi de compoziie; 1839introducerea noiunii de integrale multiple(Dirichlet); 1840- este dat o form a eliminantului a dou ecuaii algebrice de ctre James Sylvester(1814-1897), matematician englez; 1841descoperirea invarianilor de ctre matematicianul irlandez George Bole (1815-1864); introducerea noiunilor de margine inferioar i superioar ale unei funcii, de convergen uniform de ctre Weierstrass(1815-1897); 1843- descoperirea cuaternionilor de ctre William Hamilton (1805-1865); 1845- Teorema limit central este dat de matematicianul rus Pafnuti Cebev; 1846- Legea numerelor mari Cebev; introducerea variabilei complexe n teoria numerelor imaginare de ctre DAlembert; 1847 este introdus calculul logic de George Boole, creatorul algebrei booleene; 146

este introdus noiunea de ideal de ctre Ernest Kummel(1810-1893); 1851- sunt introduse noiunile de rang i signatur a unei forme ptratice i sunt propuse noiunile de matrice i jacobian(J. Sylvester); introducerea sufrafeelor riemann de ctre matematicianul german Bernhard Riemann(1826-1866), lui datorndu-se studiul integralei definite. 1852- introducerea segmentelor orientate AB de ctre Chasles Michael(1793-188) care a formulat i proprietile axei radicale a dou cercuri precum i a conicelor i cuadricelor. 1853- Kronecker(1823-1891) introduce notaia aij = det(aij ) ;
1854- este introdus noiunea de oscilaie ntr-un punct de ctre Riemann care creeaz o nou geometrie neeuclidian, numit geometria sferic; 1858- crearea calculului matriceal de ctre Arthur Cayley(1821-1895) matematician englez ; 1871 Dedekind introduce noiunile de corp i modul ceeace n limbajul actual exprim noiunile de subcorp i Z-submodul ale lui C. Tot el introduce mulimea ntregilor unui corp de numere algebrice, definind i idealele acestei mulimi i demonstreaz teorema fundamental de descompunere unic a oricrui ideal n produs de ideale prime; 1872introducerea structurilor de subinel i modul de ctre Dirichlet; introducerea numerelor raionale prin teturi de ctre Dedekind;

147

1873- Charles Hermite(1822-1901) demonstreaz 1 transcendena numrului e= lim(1 + ) n = 2,718281.... n n 1874- este dat denumirea de subgrup de ctre Sophus Lie(1842-1899); 1874-1897- crearea teoriei mulimilor de ctre Georg Cantor(1845-1918). El a introdus noiunile de mulime deschis, mulime nchis, mulime dens, mulime bine ordonat, mulime numrabil, punct de acumulare, punct izolat, produs cartezian, reuniune, intersecie. 1878- rezolvarea problemei celor patru culori pentru colorarea hrilor de ctre Cayley; 1880-sunt descoperite funciile automorfe de matematicianul francez Henri Poincare(1854-1912); 1882Ferdinand Lindemann(1852-1939) a demonstrat trascendena numrului =3,141592......; (un numr se numete transcedent dac nu este soluia niciunei ecuaii algebrice cu coeficieni raionali); tot el demonstreaz imposibilitatea cvadraturii cercului cu rigla i compasul; 1893- H. Weber, asociaz conceptului de corp, sensul de astzi, ca o structur cu o lege de grup aditiv i o nmulire asociativ, distributiv i n care orice element e inversabil; 1897- introducerea denumirii de inel de ctre Hilbert(1862-1943); 1899 -axiomatizarea geometriei de ctre David Hilbert; 1900introducerea axiomatic a numerelor ntregi(D.Hilbert); 1905- este introdus noiunea de distan ntre dou mulimi nchise de ctre matematicianul romn Dimitrie Pompeiu(1873-1954);

148

1910- este introdus denumirea de funcional de ctre Jacques Hadamard (1865-1963), unul din fondatorii analizei funcionale; 1912 -este descoperit noiunea de derivat areolar(Pompeiu) 1927-s-a stabilit formula Onicescu referitoare la geodezice dat de Octav Onicescu(1892-1983); 1928 -este introdus funcia areolar-conjugat de ctre matematicianul romn Miron Nicolescu(1903-1975); 1933 -introducerea funciilor convexe de ordin superior de ctre Tiberiu Popoviciu(1906-1975); 1936 -Matematicianul romn Gheorghe Mihoc(19061981) d o metod cunoscut sub numele de metoda Schulz-Mihoc, de determinare a legilor limit ale unui lan Markov; 1941 -teorema lui Moisil referitoare la geodezicele unui spaiu riemannian este introdus de Grigore Moisil(1906-1973); 1944 -este introdus n domeniul algebrei moderne noiunea de signatur de ctre matematicianul romn Dan Barbilian(1895-1961); 1950 -este introdus noiunea de - derivat de ctre Dan Barbilian; 1996 -celebra conjectur a lui Fermat este demonstrat de ctre Andrew Wiles de la institutul Isaac Newton din Cambridge. 2000 -este determinat cel mai mare numr prim 269725931, avnd dou milioane de cifre, obinut cu ajutorul a 20 de mii de calculatoare puse n reea;

149

BIBLIOGRAFIE. 1: N. Mihileanu- Istoria matematicii,vol.1,vol2., Editura tiinific i enciclopedic; Bucureti,1974/ 1981; 2. Vasile Bobancu- Caleidoscop matematic, Editura Niculesu; 3. Neculai Stanciu, 100 de probleme rezolvate. Editura Rafet; 4. Mic enciclopedie matematic, Editura Tehnic, Bucureti

150

Cuprins: Breviar teoretic............................................................5 Probleme enunuri i rezolvri Clasa a V-a.................................................................70 Clasa a VI-a................................................................81 Clasa a VII-a .............................................................92 Clasa a VIII-a ...........................................................116 Probleme propuse.................................................... 133 Istoricul noiunilor matematice............................... 141

152

You might also like